Está en la página 1de 72

MINISTERIO DE EDUCACIN

CURSO DE POSGRADO PARA PROFESORES DE EDUCACIN MEDIA

TEORA DEL NMERO


EQUIPO DE DISEO:
AARN ERNESTO RAMREZ FLORES
ERNESTO AMRICO HIDALGO CASTELLANOS
CARLOS MAURICIO CANJURA LINARES
CARLOS ERNESTO GMEZ RODRGUEZ
CLAUDIA PATRICIA CORCIO
HUMBERTO ALFONSO SERMEO VILLALTA
JUAN AGUSTN CUADRA
DIMAS NO TEJADA TEJADA
OSCAR ARMANDO HERNNDEZ MORALES
SIMN ALFREDRO PEA
CIUDAD UNIVERSITARIA, NOVIEMBRE DE 2010

ndice
I

Divisibilidad en Z

1. Divisibilidad en Z
1.1. Mltiplos de un entero relativo . . . . . . . . . . . . . . . . . . . . . . . . . . . . . .
1.2. Relaciones de divisibilidad en Z . . . . . . . . . . . . . . . . . . . . . . . . . . . . . .
1.3. Propiedades de la divisibilidad en Z . . . . . . . . . . . . . . . . . . . . . . . . . . .

4
4
4
5

2. La Divisin Euclidiana
2.1. La Divisin Euclidiana en N . . . . . . . . . . . . . . . . . . . . . . . . . . . . . . . .
2.2. La Divisin Euclidiana en Z . . . . . . . . . . . . . . . . . . . . . . . . . . . . . . .

7
7
9

Sistemas de Numeracin
Introduccin . . . . . . . . . . . .
Otros sistemas de numeracin . . .
Problemas de cambios de base . .
Bsqueda de la base de un sistema

II

.
.
.
.

.
.
.
.

.
.
.
.

.
.
.
.

.
.
.
.

.
.
.
.

MCM, MCD

.
.
.
.

.
.
.
.

.
.
.
.

.
.
.
.

.
.
.
.

.
.
.
.

.
.
.
.

.
.
.
.

.
.
.
.

.
.
.
.

.
.
.
.

.
.
.
.

.
.
.
.

.
.
.
.

.
.
.
.

.
.
.
.

.
.
.
.

.
.
.
.

.
.
.
.

.
.
.
.

.
.
.
.

.
.
.
.

.
.
.
.

.
.
.
.

.
.
.
.

12
12
12
13
14

19

1. Mximo Comn Divisor


19
1.1. El Mximo Comn Divisor (MCD) . . . . . . . . . . . . . . . . . . . . . . . . . . . . 19
1.2. Resultados inmediatos. . . . . . . . . . . . . . . . . . . . . . . . . . . . . . . . . . . . 19
2. Mximo Comn Divisor (continuacin)
21
2.1. Investigacin del MCD. Algoritmo de Euclides. . . . . . . . . . . . . . . . . . . . . . 21
2.2. Un resultado importante . . . . . . . . . . . . . . . . . . . . . . . . . . . . . . . . . . 22
3. Teorema de Bezout
24
3.1. Definicin . . . . . . . . . . . . . . . . . . . . . . . . . . . . . . . . . . . . . . . . . . 24
3.2. Caracterizacin: El Teorema de Bezout . . . . . . . . . . . . . . . . . . . . . . . . . . 24
4. Caractersticas y Propiedades del MCD
27
4.1. Caractersticas y Propiedades del MCD . . . . . . . . . . . . . . . . . . . . . . . . . 27
4.2. Resultado: Propiedad multiplicativa del MCD . . . . . . . . . . . . . . . . . . . . . . 27
5. Aplicaciones
28
5.1. Teorema de Gauss . . . . . . . . . . . . . . . . . . . . . . . . . . . . . . . . . . . . . 28
5.2. Fracciones irreducibles . . . . . . . . . . . . . . . . . . . . . . . . . . . . . . . . . . . 29
6. Mnimo Comn Mltiplo
30
6.1. Definicin . . . . . . . . . . . . . . . . . . . . . . . . . . . . . . . . . . . . . . . . . . 30
6.2. Resultados fundamentales . . . . . . . . . . . . . . . . . . . . . . . . . . . . . . . . . 30
6.3. Caracterizaciones . . . . . . . . . . . . . . . . . . . . . . . . . . . . . . . . . . . . . 31

Algoritmo para calcular los coeficientes de las relaciones de Bezout


34
Determinar u y v a travs de un ejemplo numrico . . . . . . . . . . . . . . . . . . . . . . 34
Generalizaciones . . . . . . . . . . . . . . . . . . . . . . . . . . . . . . . . . . . . . . . . . 34
Solucin de la ecuacin au + bv
Presentacin del problema . . .
Estudio de un caso particular .
Caso general . . . . . . . . . .

III

=c
34
. . . . . . . . . . . . . . . . . . . . . . . . . . . . . . . . . 34
. . . . . . . . . . . . . . . . . . . . . . . . . . . . . . . . . 35
. . . . . . . . . . . . . . . . . . . . . . . . . . . . . . . . . 35

Las Congruencias.

44

Actividad 1. El lenguaje de las Congruencias . . . . . . . . . . . . . . . . . . . . . . . . .


Actividad 2. Los das de la semana . . . . . . . . . . . . . . . . . . . . . . . . . . . . . . .
Actividad 3. Restos de Potencias de un entero en una divisin . . . . . . . . . . . . . . . .

44
46
47

1. Congruencias mdulo m.
47
1.1. Definiciones Equivalentes. . . . . . . . . . . . . . . . . . . . . . . . . . . . . . . . . . 47
1.2. Reglas de clculo de las Congruencias. . . . . . . . . . . . . . . . . . . . . . . . . . . 48
2. Aplicacin de las Reglas de Clculo de las Congruencias.

49

3. Resolucin de la ecuacin ax b (mod m)

50

Trabajo Dirigido: Criterios usuales de divisibilidad.

52

IV

62

Los nmeros primos

1. Definicin. Los nmeros primos


62
1.1. Definicin y existencia . . . . . . . . . . . . . . . . . . . . . . . . . . . . . . . . . . . 62
1.2. Todos los nmeros primos . . . . . . . . . . . . . . . . . . . . . . . . . . . . . . . . . 63
2. Descomposicin en productos de los nmeros primos.
2.1. Teorema Fundamental . . . . . . . . . . . . . . . . . . . . . . . . . . . . . . . . .
2.2. Investigando los divisores de un nmero natural no primo . . . . . . . . . . . . .
2.3. El Mximo Comn Divisor . . . . . . . . . . . . . . . . . . . . . . . . . . . . . .
2.4. LAS POTENCIAS DE UN NMERO Y LA DESCOMPOSICIN CANNICA.

.
.
.
.

.
.
.
.

63
63
64
64
64

3. Nmeros primos y divisibilidad en N


65
3.1. Divisibilidad por un nmero primo . . . . . . . . . . . . . . . . . . . . . . . . . . . . 65
3.2. El Pequeo Teorema de Fermat. . . . . . . . . . . . . . . . . . . . . . . . . . . . . . 65
3.3. Ejercicios de Nmeros Primos. . . . . . . . . . . . . . . . . . . . . . . . . . . . . . . 66

Parte I

Divisibilidad en Z
1.

Divisibilidad en Z

1.1.

Mltiplos de un entero relativo

Definicin: Diremos que el entero m es un mltiplo de el entero b, si existe un entero c tal que
m = bc.
Por ejemplo, los mltiplos de 3 son todos los nmeros 3c, con c un entero; as -12 y 342 son
mltiplos de 3 ya que 12 = 3(4) y 342 = 3(114).

1.2.

Relaciones de divisibilidad en Z

Definicin: Un entero b divide a otro entero a siempre y cuando exista un entero c tal que a = bc.
De manera equivalente, se dir que a es un mltiplo de b, o tambin que b es un divisor de a, o
que a es divisible por b.
Notas:
Todo nmero divide a 0, pero 0 no divide a ningn nmero.
1 y -1 dividen a todos los enteros.
Ejemplos:
1. 3 divide a 54 ya que 54 = (3)(18). Los enteros 3 y 18 son divisores de 54.
2. -5 divide a 45 por que 45 = 5(9); -5 divide a 45 por que 45 = 5(9).
3. a) Encontrar todos los divisores en N de 21
b) Encontrar todos los pares (a, b) de nmeros naturales N tales que a2 b2 = 21.
Solucin
a) De acuerdo a la definicin, 21 lo podemos escribir como 21 = 1 21, as como 21 = 3 7, y
cualquier otra forma de escribir como producto de dos (o ms) nmeros tiene, necesariamente
cualquiera de estas dos formas, luego los divisores son 1, 3, 7 y 21.
b) Recordemos que a2 b2 = (a + b)(a b) de aqu que (a + b)(a b) = 21 y como de acuerdo al
literal anterior las nicas formas de escribir como factor a 21 son 1 21 y 3 7 as:
a + b = 21 y a b = 1 luego (10, 11) es un par y como tambin a + b = 7 y a b = 3 luego (5,2) es
otra posibilidad, y como los pares deben ser naturales, estas son las nicas posibilidades.

1.3.

Propiedades de la divisibilidad en Z

a) Si b divide a a entonces b tambin lo divide.


Prueba.
Como b divide a a, esto significa que existe c tal que a = bc, pero esto tambin puede escribirse
a = (b)(c) de esto se tiene que b tambin divide a a.

b) Si b divide a a y si a 6= 0, entonces |b| |a|


Prueba.
Como b divide a a, esto significa que existe c tal que a = bc, y esto implica que
|a| = |bc| |a| = |b| |c|, luego como |c| =
6 0 se tiene que |b| |a|.
Observacin: Cuando a y b son naturales y a 6= 0, si b divide a a, entonces b a. Pero cuando a y
b son de signo cualesquiera, ese resultado no es cierto.

c) Si a divide a b y b divide a a, entonces a = b o a = b; (a 6= 0, b 6= 0).


Prueba.
Como por hiptesis a divide a b por la propiedad b) |a| |b| y como tambin b divide a a de
nuevo por la propiedad b) |b| |a|, luego esto es cierto solamente en el caso que |a| = |b|.

d) Si a divide a b y si b divide a c, entonces a divide a c.


Prueba.
Por hiptesis podemos decir que b = ak1 y c = bk2 ; con k1 y k2 enteros, luego
c = bk2 = c = (ak1 )k2 y como el producto de dos enteros, es otro entero, entonces c = a(k1 k2 ) de
donde se deduce que a divide a c.

e) Si a divide a b y c, entonces a divide a b + c, b c, y de manera general divide a todo entero de


la forma bx + cy, con x e y enteros.
Prueba.
Por hiptesis podemos decir que b = ak1 y c = ak2 ; con k1 y k2 enteros. Si d = bx + cy, con x e y
enteros, entonces:
d = ak1 x + ak2 y = d = a(k1 x + k2 y); puesto que (k1 x + k2 y) es un entero, podemos concluir
que a divide a d.
5

Si hacemos x = y = 1, entonces d toma la forma b + c y como a divide a d, podemos concluir que


a divide a b + c.
De al misma forma haciendo x = 1 y y = 1, puede concluirse que a divide a b c.

f ) Si a divide a b, entonces ac divide a bc, para cualquier entero c.


Prueba.
Como Si a divide a b, esto de acuerdo a la definicin significa que b = ak1 , luego
bc = ak1 c = bc = ac(k1 ) y se tiene el resultado.

UNA PROPIEDAD NOTABLE DEL CONJUNTO N.


Los subconjuntos de N poseen la propiedad siguiente: Todo subconjunto E de enteros
estrictamente positivos posee un mnimo.
Por ejemplo si E es el conjunto de nmeros de la forma 2n + 5, con n 0, n Z el elemento
mnimo de E es el 5.
Esta propiedad no es cierta en todo Z. Veamos por ejemplo que si tomamos los nmeros menores
que 20 notamos que no hay elemento mnimo.

COMENTARIO
Para trabajar la hiptesis b divide a a escribimos a = bq con q Z, ya que una igualdad
facilita los clculos.
Observacin: Podemos ver de lo anterior que b no nulo divide a se traduce en la expresin
a
= q con q entero. Esto es evidentemente equivalente a escribir a = bq.
b
Lo esencial en las dos formas de escribir esta expresin es dejar claro que q debe ser entero.
Para interpretar una igualdad entre nmeros enteros, pensemos que la hiptesis p = mq con m y q
enteros, significa que m divide a p y tambin q divide a p.

EJEMPLOS RESUELTOS. Como utilizar la divisibilidad para encontrar los divisores comunes a
dos nmeros enteros:
Ejemplo 1. Si k es un nmero natural, a = 9k + 2 y b = 12k + 1. Pruebe que los nicos divisores
positivos posibles a los enteros a y b son 1 y 5.
Solucin
6

Sea d un divisor comn de a y b. (Dos enteros siempre tienen por lo menos un divisor comn: el 1.
Por esta razn es posible comenzar dndonos un nmero d divisor comn de a y b, sin justificar la
existencia de tal divisor). Para encontrar los valores posibles de d, un mtodo consiste en utilizar
la propiedad e) que establece que si d divide a a y b tambin divide a todo entero de la forma
ua + vb con u y v enteros cualesquiera. As d divide a u(9k + 2) + v(12k + 1).
Este mtodo se consigue dando valores a u y v con el fin de eliminar k. As por ejemplo escogiendo
u = 4 y v = 3 tenemos que d divide a 36k + 8 36k 3 lo cual vale 5. Luego los valores posibles
de d son 1 y 5.

Observacin: La eleccin de u = 4 y v = 3 que nos condujo a 36k y 36k, es la mejor por que 36
es el mnimo comn mltiplo de 9 y 12.
Si hubisemos elegido u = 12 y v = 9 se introduce un factor 3 adicional
Ejemplo 2. Sea k un nmero natural, a = 13k + 1 y b = 26k + 4. Probar que los divisores
positivos y comunes a a y b son 1, 2, 3 y 6.
Solucin:
De nuevo por la propiedad e) como en el ejemplo anterior, d divide a u(13k + 1) + v(26k + 4).
En este caso 26 es mltiplo de 13 haciendo u = 2 y v = 1, tendramos que
d |26k + 2 + (26k) + 4 d|6 , es decir, d puede tomar los valores de 1, 2, 3 y 6.

2.

La Divisin Euclidiana

Uno de los procesos algortmicos, que en matemtica mayor dificultad representa en el proceso
de aprendizaje temprano de la matemtica, es el de la divisin, el cual aunque hemos aprendido,
en muchos casos no tenemos idea de cual es el proceso matemtico que est inmerso en el. En este
caso se dar por descontado que tal algoritmo en la manera tradicional, es conocido y se tratar de
ir un poco mas all, dada la importancia de este proceso en teora de nmeros.

2.1.

La Divisin Euclidiana en N

Si queremos dividir a un nmero b entre otro nmero a, bsicamente lo que tratamos de hacer
es aproximar de manera lo mejor posible a b por un mltiplo de a, la diferencia entre b y ese
mltiplo de a es lo que llamaremos el resto de la divisin, que ser nulo exactamente en el caso de
que b sea mltiplo de a. Precisemos un poco acerca de todo esto.
Siendo a un nmero natural, la secuencia 0, a, 2a, 3a, ... de mltiplos no negativos de a es
estrictamente creciente; luego, alguno de ellos ser mayor que b y siempre podemos elegirlo de tal
manera que el anterior no supere a b (Por el principio de buena ordenacin de los nmeros
naturales). Esto es, existe exactamente un entero no negativo q tal que:
qa b < (q + 1)a

Utilizando un lenguaje un poco mas informal, observemos que no hemos hecho otra cosa que
ubicar a b entre dos mltiplos consecutivos de a. El siguiente esquema ilustra tal situacin:

Puesto que dos mltiplos consecutivos de a difieren en a, es claro entonces que la diferencia b qa,
que designaremos por r, es menor que a. En resumen, quedan determinados dos enteros q y r
verificando las dos condiciones siguientes:
b = qa + r
0r<a
La segunda condicin es a la que nos referamos cuando hablbamos de manera lo mejor posible.
Concretamente, qa es el mltiplo de a mas cercano a b entre los que son menores o iguales que b.
Para finalizar , veamos que q y r son los nicos enteros que verifican las dos condiciones
anteriores. Supongamos para ello que p y s son otro par de nmeros que satisfacen las dos
condiciones anteriores, es decir que b = pa + s, siendo s alguno de los nmeros 0, 1, 2, ..., a 1. Si
fuera por ejemplo que p > q (el caso p < q es similar), restando las dos igualdades tenemos:
0 = (p q)a + (s r) = (p q)a = r s.
En consecuencia, r s es positivo y adems es mltiplo de a, pero por otro lado, la condicin del
residuo nos establece que r s r < a, lo cual es posible solamente si r s = 0 = r = s y esto a
su vez establece la igualdad entre p y q.
Acabamos de demostrar un teorema muy importante en la teora de nmero:
Teorema. Dados dos nmeros naturales a y b, existe un nico par de enteros q y r (que
llamaremos el cociente y el resto de la divisin entera de a por b, respectivamente) satisfaciendo
las dos siguientes condiciones:
a = bq + r y 0 r < b
Definicin: Efectuar la divisin euclidiana en N de un nmero natural a por el nmero natural b,
b 6= 0 consiste en encontrar los pares de naturales (q, r) tales que a = bq + r y 0 r < b.
Nota: El residuo o resto es estrictamente menor que el divisor.
Ejemplo. La divisin euclidiana de 114 por 8 es 114 = 8 14 + 2, 0 2 < 8.
Ejemplo. Encontrar un nmero natural que al dividirse por 23 nos de resto 1 y que al dividirse
por 17 nos da el mismo cociente y por residuo 13.
8

Solucin
Sea n el nmero natural buscado, luego por la primera condicin del problema n puede escribirse:
n = 23q + 1 y por la segunda condicin: n = 17q + 13, lo cual nos da un sistema de dos ecuaciones
con dos incgnitas, el cual es resoluble, por ejemplo utilizando el mtodo de igualacin, luego:
23q + 1 = 17q + 13
23q 17q = 13 1
6q = 12
q=2
Y sustituyendo este valor en cualquiera de las dos ecuaciones originales: n = 23(2) + 1 y de esto
resulta n = 47, que nos da el nmero buscado, el cual es nico en N.
Ejemplo. Encuentre todos los nmeros naturales N tales que n + 1 | n2 + 1.
Solucin.
Este problema nos brinda la posibilidad de utilizar el algoritmo de la divisin en un contexto muy
interesante, tratando de construir el divisor; veamos:
n + 1 = n 1 + 2 = n + 1 = (n + 1)(n 1) + 2, luego por la propiedad e) n + 1 debe dividir a
la suma, esto obliga a que n + 1 divida a 2, y los nicos divisores de este nmero son el 1 y el
mismo (mas adelante se definir de manera formal el concepto de nmero primo) es decir
n + 1 = 1 o n + 1 = 2 y de aqu se deduce que n solamente puede valer 1.
Observacin: Decir que b divide a a, es equivalente a decir que la divisin euclidiana de a por b el
resto es nulo.
Ejercicio: Utiliza la idea del ltimo ejemplo, para resolver: Si 7 | (3x + 2) pruebe que
7 | (15x2 11x 14).

2.2.

La Divisin Euclidiana en Z

La divisin euclidiana se generaliza a dos nmeros enteros cualesquiera, a travs del siguiente
resultado:
a y b son dos nmeros enteros cualesquiera y b es no nulo. Entonces existe un nico entero q y un
nico entero positivo r, tales que a = bq + r y 0 r < |b|.
Para comprender ms.
Que condicin debe cumplir la igualdad a = bq + r, entre dos nmeros naturales para que
necesariamente pueda ser traducida en la divisin euclidiana de a por b?
La condicin 0 r < b es indispensable para que la igualdad a = bq + r sea equivalente a la
divisin euclidiana.
9

Comentario: una consecuencia fundamental del primer teorema es la siguiente:


Si b es un nmero estrictamente positivo, se puede expresar cualquier nmero natural en funcin
de b y de sus restos posibles de su divisin euclidiana por b
En efecto, los posibles restos de dividir por b cualquier nmero son 0, 1, 2, ..., b 1 as todo nmero
natural n puede escribirse n = bq + r con r = 0 1 2,..., b 1.
As por ejemplo todo nmero natural n puede escribirse como 6p 6p + 1 6p + 2 6p + 3
6p + 4 6p + 5 con p un nmero natural.
Todo nmero natural n puede escribirse como 2p (Que le diremos par) como 2p + 1 (que
llamaremos impar) con p un nmero natural.
Ejemplo 1. Si a = 37, b = 11 escribimos 37 = 11 3 + 4 que es el mismo caso de la divisin
euclidiana en N, luego podemos escribir 37 = (11) (3) + 4 (Ntese que q = 3 y r = 4,
r < |11|)
Ejemplo 2. Si a = 37, b = 11 de nuevo escribimos 37 = 11 3 + 4 que es el mismo caso de la
divisin euclidiana en N, luego podemos escribir 37 = (11) (3) 4 , pero hemos definido el
resto como un nmero positivo menor que 11, para lo cual nos valemos de reescribir en la forma:
37 = (11) (3) 4 + 11 11 y esto, operando y agrupando convenientemente se convierte en:
37 = (11) (4) + 7. (Ntese que q = 4 y r = 7, r < |11|).
Ejemplo 3. Si a = 37, b = 11 escribimos 37 = 11 3 + 4 que es el mismo caso del ejemplo 2,
37 = (11) 4 + 7. (Ntese que q = 4 y r = 7, r < |11|).
Ejemplo 4. Veamos una ilustracin de como utilizar el comentario del teorema 1 efectuado arriba.
Demostrar que para todo nmero natural n, n(n + 1)(2n + 1) es divisible por 3.
Demostracin
Sea a = n(n + 1)(2n + 1) vamos a demostrar que a tiene la forma 3q con q entero, utilizaremos el
hecho de que todo natural es de la forma 3p 3p + 1 3p + 2, esto reduce a verificar tres casos
para valores de n, es decir si n = 3p 3p + 1 3p + 2.
Veamos el primer caso:
Si n = 3p entonces a = 3p(3p + 1)(6p + 1) = 3q1 con q1 = (3p + 1)(6p + 1).
Si n = 3p + 1 entonces a = (3p + 1)(3p + 4)(6p + 3) = 3q2 con q2 = (3p + 1)(3p + 4)(2p + 1).
Y finalmente, si n = 3p + 2 entonces a = (3p + 2)(3p + 3)(6p + 5) = 3q3 con
q3 = (3p + 2)(p + 1)(6p + 5). Lo que completa la prueba
Ejemplo 5. Cules son los posibles restos de dividir un cuadrado por 4? y por 8?
Solucin.

10

De nuevo utilizaremos la tcnica descrita en el ejemplo anterior, ahora las formas posibles de
escribir un nmero natural n cuando se divide por 4 son 4p, 4p + 1, 4p + 2 y 4p + 3, pero como
estamos interesados en saber los residuos de dividir por 4 a un cuadrado, n tiene la forma n, de
nuevo veamos los casos:
Si n = 4p = n = 16p, deja residuo 0.
Si n = 4p + 1 = n = 16p + 8p + 1, deja residuo 1.
Si n = 4p + 2 = n = 16p + 16p + 4 = 4(4p + 4p + 1), deja residuo 0.
Si n = 4p + 3 = n = 16p + 24p + 9 = 4(4p + 6p + 2) + 1, deja residuo 1. Luego los nicos
residuos son 0 y 1
Utiliza la misma tcnica para responder la pregunta: y por 8?.
Ejemplo 6. La sucesin (Un ) est definida para todo entero n por Un = 32n 2n . Demostrar que
para todo n, Un es divisible por 7.
Demostracin.
Recordemos un poco el mdulo anterior, especificamente el teorema del binomio:
32n 2n = 9n 2n = (7 + 2)n 2n


Pn
n
n
Pero (7 + 2) = k=0
7k 2nk , de esta expresin es claro que cuando k = 0, que es el
k
primer trmino de la suma, este valdr 2n el cual se anula en la expresin original y todos los dems
tendrn siempre un factor 7, msexplcitamente:



Pn
Pn
n
n
(7 + 2)n 2n = 2n + k=1
7k 2nk 2n =
7k 2nk , lo que completa la
k=1
k
k
prueba.
APLICACIN
Qu da de la semana fue el 15 de abril de 1707?. (Fecha de nacimiento de Euler)
Para contestar esta pregunta es necesario recordar que en nuestro calendario un ao normal
consta de 52 semanas y 1 da, osea 365 das. Un ao bisiesto consta de un da ms, agregado al mes
de febrero, osea consta de 366 das. Un ao que no es secular (o sea que no corresponde a un siglo)
es bisiesto si y slo si es divisible por 4. Los aos seculares tales como 1600, 1700, 1800, 1900,... son
bisiestos si y slo si, son divisibles por 400. Por consiguiente, los aos 1600 y 2000 son bisiestos,
pero no lo son los aos 1700, 1800 y 1900.
La observacin fundamental para determinar en que da de la semana cay una fecha dada es
que dos fechas dadas caen en el mismo da de la semana s, y slo s, el nmero de das del intervalo
que forman esas dos fechas tienen resto 1 en la divisin por 7.
As por ejemplo, si el 1 de enero fue lunes y el ao no es bisiesto, del 1 de enero al 31 de diciembre
hay 365 das; esto es 365 = 52 7 + 1. Por lo tanto, ambas fechas caen el mismo da de la semana. Si
el ao fuese bisiesto sera 366 = 52 7 + 2, y en consecuencia el 31 de diciembre caera el siguiente
da de la semana al que corresponde al 1 de enero.
Con esta breve explicacin es posible calcular el da de la semana que le correspondi al natalicio
de Euler. La informacin bsica se obtiene de un calendario 2010 que tenemos a mano.
11

Aos transcurridos de 1707 a 2010: 2010-1707=303 aos


Aos bisiestos intermedios: estos van de 1708 a 2008, osea 2008-1708=300, pero 1800
y 1900

+
1

2
= 74
no fueron bisiestos, por tanto el nmero de aos bisiestos intermedios fue 300
4
Del 15 de abril de 1707 al 15 de abril de 2010 han transcurrido 303 aos ms 1 da.
Por tanto el nmero total de das fue: 303 365 + 74 + 1 = 110670 = 15810 7
Dado que el 15 de abril de 2010 fue jueves, se puede concluir que el 15 de abril de 1707 fue viernes,
As el gran matemtico Leonhard Paul Euler naci un viernes. (Recordemos que en este conteo, el
da conocido es el 15 de abril de 2010 que es jueves, si empezamos el conteo a partir de este da
pero hacia atrs, vemos que va as: jueves, mircoles, martes, lunes, domingo, sbado y viernes que
sera el mltiplo de 7, y si seguimos esta secuencia 15810 veces obtenemos el da buscado).

Sistemas de Numeracin
1

Introduccin

Proponer un sistema de numeracin es dar un mtodo que permite escribir todos los nmeros
enteros con un nmero finito de smbolos. Nuestro sistema usual de numeracin es el decimal, este
permite escribir todos los nmeros enteros utilizando diez cifras (dgitos) que son 0, 1, 2, 3, 4, 5, 6,
7, 8, 9.
Comentario: casi todas las antiguas civilizaciones de China, Mesopotamia, Egipto, Amrica
del Sur (mayas, aztecas ,...), tuvieron su sistema de numeracin. Estos sistemas no permiten un
clculo fcil de las sumas, productos, cocientes. En el sistema de numeracin Romano, los resultados
de las operaciones debe ser aprendidos por medio de ejemplos de clculo, 11x53, es decir, XI.LIII
en nmeros romanos; esto porque, no existe ninguna regla fcil (no tiene ninguna base) para llegar
a 583, es decir, DLXXXIII.
Nuestro sistema decimal tiene la gran ventaja de hacer todas las operaciones simples y esto
mediante la creacin de la cifra cero, que fue transmitida por los rabes en Occidente alrededor
del ao 700, as, cada dgito tiene un valor diferente segn la posicin en la representacin de un
nmero.

Otros sistemas de numeracin

En nuestro sistema decimal, todo entero es escrito an 10n + an1 10n1 + . . . + a1 10 + a0 , donde
cada ai es un nmero estrictamente menor que 10 (por lo tanto de una cifra). As,
3612 = 3 103 + 6 102 + 1 10 + 2.
Dejamos como ejercicio probar que: si elegimos un entero a positivo, todo entero m es escrito
de manera nica m = u1 an + un1 an1 + . . . + u1 a + u0 , donde cada ui es un entero positivo menor
que a.
Decimos que esta representacin es la escritura de m en el sistema de base a, y que los nmeros
ui son las cifras en dicho sistema. Se escribe de forma compacta: m = un un1 un2 . . . u1 u0
m = un un1 un2 . . . u1 u0 .

12

(la barra indica que no es el producto de los nmeros).


Ejemplos:
1. Sistema de base dos o binaria: la base es dos, las cifras son 0,1. Todo nmero es escrito
con estas dos cifras 0 y 1. Ejemplos: cero 0, uno 1, dos 10, tres 11, cuatro 100.
2. Sistema base ocho u octal: la base es ocho, las cifras son 0, 1, 2, 3, 4, 5, 6, 7. Ejemplos:
cero 0, uno 1, dos 2, tres 3, cuatro 4, cinco 5, seis 6, siete 7, ocho 10, nueve 11.
3. Sistema base doce o duodecimal: la base es doce. las cifras son 1, 2, 3, 4, 5, 6, 7, 8, 9,, .
Hay que aadir dos smbolos y a las cifras habituales, por diez, por once, porque el
nmero doce se escribe 10 en este sistema, el nmero trece se escribe 11...
Comentario: el sistema binario es perfectamente adaptado en la tecnologa actual, calculadoras o
computadoras donde se emplea elementos que pueden tomar uno de los dos estados fsicos apagado
y encendido, apagado lo representamos con 0 y encendido con 1.
El hecho que el tamao en la escritura de los nmeros en base dos va aumentando rpidamente,
ha llevado a los informticos a hacer uso intenso del sistema hexadecimal (base 16). Por ejemplo,
en el sistema de codificacin ASCII, la letra A es codificada en base 2 por 1000001 mientras que en
hexadecimal su cdigo es 41.

Problemas de cambios de base

Escribir en el sistema decimal un nmero escrito en un sistema de base a.


Ejemplo: el nmero N se escribe 10011001 en el sistema binario. Cul su escritura en el sistema
decimal?
Aplicando los resultados del apartado anterior, a partir de la ltima cifra de la derecha son las
unidades. N = 1 27 + 0 26 + 0 25 + 1 24 + 1 23 + 0 22 + 0 2 + 1, luego N , en decimal
es el nmero 1 + 22 + 24 + 27 , es decir 153.
Ejercicios: En cada caso, dar la escritura decimal de N.
1. La escritura de N en base 2 es 111000110.
2. La escritura de N en base 8 es 76152
3. La escritura de N en base 12 es 7645.
Escribir en un sistema de base a un nmero escrito en el sistema decimal.
Ejemplo. El nmero N = 2183 est escrito en el sistema decimal. Cul es su escritura en el
sistema base ocho?.
Aplicando los resultados del apartado anterior, escribamos N en su forma de base ocho: N =
8n un + . . . + 8u1 + u0 , cada uno de los nmeros ui son cifras de 0 a 7. Vemos que N = 8N + u0 ,
donde u0 es el resto de la divisin eucldea de N por 8, es decir, 2183 = 8 272 + 7, donde u0 = 7.
Para lograr la forma que deseamos, hay que dividir a 272 por 8, lo que da: 272 = 8 34 + 0, as
que, N = 8 (8 34 + 0) + 7 = 82 34 + 8 0 + 7 y u1 = 0.
(Tenga en cuenta que se sustituye 272 por 8 34 + 0, y no por 8 34, porque llevara a un
resultado falso). La divisin de 34 por 8 es 34 = 8 4 + 2, as que: N = 82 (8 4 + 2) + 8 0 + 7 =
83 4 + 82 2 + 8 0 + 7. La escritura de N en base ocho en entonces 4207.
Ejercicios:
13

1. Escribir en base doce, los nmeros once, trece, diecinueve, ciento veinte y cinco.
2. El nmero N = 6215 est escrito en el sistema decimal. Cul es su escritura en base ocho?
3. El nmero N = 13615 est escrito en el sistema decimal. Cul es su escritura en base siete?
En base doce?
Escribir en el sistema base a nmero N escrito en base b.
En primer lugar hay que escribir a N en el sistema decimal, y luego pasar la escritura en decimal
a base b. El nmero N = 5012 est escrito en base 7. Cul es su escritura en base dos?.

Bsqueda de la base de un sistema

Ejemplo: el nmero N se escribe 23 en el sistema decimal y se escribe 27 en un sistema base a.


Podemos encontrar a?. Observemos en primer lugar que a es mayor que 7 porque 7 es un dgito
del sistema de base a. Justificar la igualdad 23 = 2 a + 7. Deducir que a es el nmero ocho.
Ejercicios.
1. El nmero N se escribe 136 en decimal y se escribe 253 en base a. Se puede encontrar a?
2. El nmero N se escribe 303 en decimal y se escribe 523 en base a. Se puede encontrar a?

14

3.

Ejercicios
1. Escriba todos los divisores en Z de los nmeros 20, 28 y 75.
2. a y b son enteros, b 6= 0. Qu condicin debe cumplir el nmero

a
b

para que sea un entero?.

3. n es un entero natural mayor o igual a 2 y A = n4 1.


a) Demostrar que n 1, n + 1 y n2 + 1 son divisores de A.
b) Deducir otros divisores de A.
4. Encuentre todos los divisores en N de 21. Encuentre todos los pares de nmeros naturales
(a, b) tales que a2 b2 = 21.
5. Encuentre todos los pares de nmeros naturales (x, y) tales que x2 2xy = 15.
6. k es un natural, a = 3k + 5 y b = 2k + 1. Pruebe que los posibles divisores positivos comunes
de a y b son 1 y 7.
7. Encuentre todos los enteros n tales que n + 8 es un mltiplo de n.
8. Explique porque es imposible encontrar u y v en Z tales que 6u 9v = 2.
9. Encuentre todos los enteros n que divididos por 4 tengan cociente igual a su resto.
10. Encuentre un natural que dividido por 23 de como resto 1 y que dividido por 17 d el mismo
cociente con un resto de 13.
11. b es un natural no nulo. La divisin eucldea de 990 por b, da como cociente 39.
a) Escriba la relacin que traduce esta divisin.
b) Pruebe que 39b 990 < 40b. Deduzca b y r.
12. Demuestre las siguientes propiedades:
a) La suma de dos nmeros impares consecutivos es mltiplo de 4.
b) La suma de tres nmeros impares consecutivos es divisible por 3 pero no por 6.
c) El producto de dos nmeros pares consecutivos es divisible por 8.
13. Las edades de tres personas son nmeros consecutivos. Si se dividen dichos nmeros por 9,
dos de ellos tienen cociente 6 y el restante 7 Que edades tienen?
14. Supongamos que al dividir por 12 catorce nmeros enteros consecutivos se obtienen tres
cocientes diferentes Cules son los restos de dividir por 12 el menor y mayor de dichos
nmeros?
15. Dos nmeros enteros m y n (m < n) difieren en 110. El resto de dividir m por 9 es mayor que
el de dividir n por 9, y ninguno de los dos es mltiplo de 9. Cules son esos restos?
15

16. Sean a, b y c enteros tales que a2 + b2 + c2 = n con n no mltiplo de 3. Demuestre que al


menos uno de los tres nmeros a, b y c es mltiplo de 3.
17. El cociente de un entero cualquiera x al dividirlo por 3 es 7. Cules son los restos posibles?
Deduzca cuales son los valores posibles para x.
18. La suma de dos nmeros enteros a y b es 444. La divisin eucldea de a por b tiene como
cociente 4 y como resto 24. Determine a y b.
19. La diferencia de dos nmeros enteros a y b es a b = 996. La divisin eucldea de a por b
tiene como cociente 4 y como resto 60. Encontrar a y b.

20. a es un entero cualquiera. Demostrar que a a2 1 es un mltiplo de 2 y de 3.
21. n es un entero. Demostrar que para cualquier n, 3n2 + 5n + 1 es impar y deducir que nunca
es divisible por n (n + 1).
22. a y b son dos enteros cualquiera. Demostrar que si a2 + b2 es divisible por 7, entonces a y b
son divisibles por 7.
23. Encuentre a, b y c tal que ab + bc + ac = abc, con 0 < a < b < c.

2
24. a es un entero, a 1, y A = a2 + (a 1)2 . Demostrar que el resto de dividir A por 4a2 es
2
(2a 1) .
25. n es un entero. Demostrar que si n > 6, el nmero 6n tiene por lo menos 8 divisores.
26. p es un entero estrictamente positivo. Cuntos son los naturales que estn estrictamente
entre dos mltiplos de p? Deducir que entre p naturales consecutivos cualesquiera, hay por lo
menos uno mltiplo de p.
27. Si n es un entero natural tal que n 1, y definimos An = (n + 1) (n + 2) (2n 1) (2n).
Pruebe por recurrencia que An es divisible por 2n .
28. Demostrar que si p es impar, la suma de p nmeros consecutivos es un mltiplo de p.
29. Para cualquier entero no nulo, definimos: M = 9n + 1y N = 9n 1. Probar que 81n2 1 es
divisible por 4 cuando n es impar y slo en este caso.
30. Si n 2 y k es positivo, entonces (n 1)2 divide a nk 1 si y slo si n 1 divide a k.
31. Para todo entero positivo n, el nmero An = 5n + 2 3n1 + 1 es mltiplo de 8.
32. Cul es el mayor entero positivo n tal que (n + 10)|(n3 + 100)?
33. Si m y n son dos nmeros impares, entonces
a) 8 divide a m2 n2 .
b) 8 divide a m4 + n4 2.
34. n es un natural. Demuestre que para cualquier n, los nmeros n3 n y n (n + 1) (2n + 1) son
divisibles por 2 y por 3.
16

35. Encuentre todos los pares de enteros (x, y) tales que x2 y 2 = 35.
36. Encuentre todos los pares de enteros tales que la suma es un mltiplo de su producto.
37. Encuentre todos los enteros tales que: xyz = 4 (x + y + z), donde 0 < x y z.
38. Sean a1 , a2 , . . . , an nmero enteros tales que a1 a2 an = n y a1 +a2 +. . .+an = 0. Demostrar
que 4|n.
39. Si an = 6n + 8n , determinar el resto al dividir a1991 entre 49.
40. Sea a1 , a2 , . . . , an una permutacin arbitraria de los nmeros 1, 2, . . . , n. Probar que, si n es
impar, el producto (a1 1)(a2 2) (an n) es un nmero par.
41. Probar las siguientes propiedades
a) Demostrar por recurrencia que para cualquier natural n, 23n 1 es divisible por 7.
b) Deducir que 23n+1 2 es un mltiplo de 7 y que 23n+2 4 es un mltiplo de 7.
c) Determine los restos de la divisin por 7 de las potencias de 2.
42. La sucesin (un ) es definida para todo natural n por un = 5n3 + n.
a) Verifique que: un+1 un = 3[5n(n + 1) + 2].
b) Demostrar que para todo entero n, 5n(n + 1) + 2 es nmero par.
c) Demostrar por recurrencia que para todo entero n, un es divisible por 6.
43. En una divisin eucldea el dividendo tiene p cifras y el divisor q cifras. De cuntas cifras es
el cociente?
44. a y b son dos enteros. En la divisin eucldea de a por b, el cociente es no nulo. Pruebe que a
es mayor que el doble del resto.
45. En la divisin eucldea entre dos enteros positivos, el dividendo es 1517 y el cociente 75. Qu
puede decirse del divisor y el resto?
46. El dividendo en una divisin es menor a 300. El cociente es 72 y el resto es 12. Buscamos el
divisor y el dividendo. Explique por qu no hay una solucin.
47. Efectuamos la divisin eucldea de n por 4 y llamamos al cociente q y r al resto.
a) Escriba la relacin que traduce esta divisin.
b) Dado n, le asociamos su resto r y por tanto nos da una sucesin.
1) Mostrar que dicha sucesin es peridica.
2) Represente grficamente esta sucesin cuando n toma los valores que estn en
[12, 11].
48. Sean a y b naturales. Efectuamos la divisin eucldea de a por b, luego aumentamos el dividendo en 52 y el divisor en 4. El cociente y le divisor no cambian. Calcule el cociente.
17

49. La diferencia entre dos naturales es 538. Si dividimos uno por el otro, el cociente es 13 y el
resto es 34. Cules son los nmeros?. La suma de dos naturales es 2096. Si dividimos uno
por el otro, el cociente es 5 y el resto es 206. Cules son los nmeros?.
50. a y b son dos enteros. En la divisin eucldea de a por b, el resto r es mayor o igual a su
cociente q. Demostrar que si dividimos a por b + 1, obtenemos el mismo cociente.
51. a y b son dos enteros positivos. En la divisin eucldea de a por b, el cociente es q y el resto
r. Se agrega a a un nmero cualquiera h.
a) Represente en un eje a, bq, b(q + 1).
b) Para que valores de h el cociente de a + h por b sigue siendo igual a q.
c) Aplicaciones numricas:
1) a = 61, b = 17.
2) a = 240, b = 39.
52. a y b son dos enteros tales que 0 < b2 < a. Sea q el cociente y r el resto en la divisin eucldea
de a por b.
a) Escriba la relacin que traduce las hiptesis.
b) Demuestre que b c.
c) Demuestre que en la divisin de a por c, el cociente es b y el resto no cambia.
d ) Encuentre un contraejemplo que muestre que si a < b2 , puede suceder que el cociente de
a por c no sea b.
53. El nmero que se escribe 68425 en el sistema decimal. Escriba este nmero en el sistema base
2, base 8 y en base 12.
54. El desarrollo decimal del producto de dos nmeros termina en 7 y uno de ellos tiene resto 4
al dividirlo por 5. Qu resto tiene el otro?
55. Si elegimos un entero a positivo, todo entero m es escrito de manera nica m = un an +
un1 an1 + . . . + u1 a + u0 , donde cada ui es un entero positivo menor que a.
56. Encontrar que da es la fecha indicada:
a) 23 de febrero de 1855 (muri Gauss).
b) 15 de abril de 1707 (naci Euler).
c) 24 de marzo de 1980 (muri Arnulfo Romero).

18

Parte II

MCM, MCD
1.
1.1.

Mximo Comn Divisor


El Mximo Comn Divisor (MCD)

El objetivo de esta seccin es investigar los divisores positivos comunes a dos enteros positivos
a y b.
Por convencin, en esta seccin y las siguientes, cuando se hable de divisores de un nmero
natural, se referir siempre a los divisores positivos.
Divisores comunes a dos enteros positivos.
Para todo nmero natural a, denotamos D(a) al conjunto de los divisores de a.
Ejemplo. D(6) = {1, 2, 3, 6}; D(15) = {1, 3, 5, 15}; D(19) = {1, 19}.
Ejemplo. D(a) solamente contiene nmeros naturales menores o iguales que a. Siempre que a > 1,
D(a) contiene siempre a 1 y a a. El elemento ms grande de D(a) es a y el ms pequeo es 1. El
conjunto D(1) solamente contiene un elemento: el nmero 1.
Para todo par de nmeros naturales a y b, denotamos D(a, b) al conjunto de divisores comunes
de a y b.
As, D(a, b) = D(a) D(b) . Es claro que D(a, b) = D(b, a).
Ejemplo. D(6, 15) = D(6) D(15) = {1, 3}; D(15, 19) = D(15) D(19) = {1}.
Ejemplo. D(a, b) es un conjunto no vaco, siempre contiene al 1, y los nmeros que contiene son
menores o iguales a a y b. Por lo tanto D(a, b) posee un elemento mayor que se llama el Mximo
Comn Divisor de a y b.
Ejemplo. El mximo comn divisor de a y b se abrevia MCD.
Ejemplo. El MCD de 6 y 15 es 3; el de 15 y 19 es 1.
Observacin. Para conocer el conjunto de divisores negativos de a, es suficiente tomar los negativos
de los elementos de D(a).
As, el conjunto de los divisores en Z de 6 es {6, 3, 2, 1, 1, 2, 3, 6}.

1.2.

Resultados inmediatos.

1. Cualquiera que sea el nmero natural c, D(c, 0) = D(c).


En efecto, todo entero divide a 0, entonces los divisores comunes a 0 y c son los divisores de
c.
2. Si b divide a a, entonces b es el MCD de a y b.
En efecto, en este caso, todo divisor de b es un divisor de a. Entonces D(a, b) = D(b) y como
b es el elemento mayor de D(b) es tambin el elemento mayor de D(a, b).
Ejemplo. D(6, 12) = {1, 2, 3, 6} = D(6); 6 es el MCD de 6 y 12.
19

3. Si a = b es claro que D(a, b) = D(a).


Para mayor comprensin.
Cmo programar en tu calculadora un algoritmo para obtener los divisores de un nmero
natural?
Dado un natural n, el algoritmo ms simple consiste en examinar con la ayuda de un bucle
FOR todos los enteros k de 1 a n. Si k divide a n entonces lo mostramos. El algoritmo puede
ser escrito as:
Algoritmo
1. Leer n.
2. Correr el
FOR:

bucle

Para todo k de 1 a
n
Si k divide a n entonces mostrar k.
3. Fin
del
FOR.

bucle

Comentarios
1. La mquina demanda el
valor de n.
2. k va a tomar sucesivamente todos los valores
de 1 a n.
k divide a n puede
programarse verificando
n
que la parte entera de
k
n
debe es igual a .
k

Familia Casio
N:?N
For 1 K to N
If
Int(N/K)=N/K
Then KN
End If
Next N

Familia TI
Input N, N
For(K,1,N)
If
Int(N/K)=N/K
Disp K
End

Ejercicios resueltos.
Ejercicio 1. Demostrar que D(a, b) = D(b, a b).
1. a y b son dos enteros naturales, a b. Demuestre que el conjunto de divisores comunes de a
y b es el mismo conjunto de divisores comunes de b y a b.
2. Aplique varias veces este resultado para demostrar que los divisores comunes de 168 y 264
son los divisores de 24. Deduzca el MCD de 168 y 264.

20

Solucin Comentada
1. Debe demostrarse que los dos conjun- 2. Del numeral 1 se deduce que:
tos D(a, b) y D(a b, b) son iguales.
D(168, 264) = D(96, 168)
Para ello, se demostrar que todo nmero d de D(a, b) est en D(a b, b), y
Repitiendo este proceso se obtiene:
luego, que todo nmero d de D(a b, b)
est en D(a, b).
Supongamos que d est en D(a, b) enD(96, 168) = D(72, 96) = D(24, 72) = D(24, 48)
tonces d divide a a y b. De aqu resulta
que d divide a a b entonces d est en
D(a b, b).
= D(24, 24) = D(24)
Recprocamente, si d est en D(a b, b),
divide a a b y divide a b, y tambin d Los divisores comunes de 168 y 264 son los divisores
divide a la suma de estos nmeros, por de 24. El mayor es entonces 24.
lo que d divide a a y b y d D(a, b).
Un poco de historia: Es por este mtodo de sustracResulta entonces que D(a, b)= D(a ciones y no por el de divisiones sucesivas (el cual se
b, b).
explica e la siguiente seccin) que Euclides, en su libro
Los Elementos, calcula el MCD de dos nmeros. Este
mtodo conocido por el nombre de Antiferesis, que
significa accin de restar alternadamente, parece que
ya era conocido por los Pitagricos.
De hecho, Euclides utiliz los nmeros nicamente como segmentos de recta (para los griegos de esa poca,
el nmero es una medida), lo cual le permiti tratar los
nmeros de forma general an sin conocer los valores
precisos de estos.

2.
2.1.

Mximo Comn Divisor (continuacin)


Investigacin del MCD. Algoritmo de Euclides.

En la subseccin 1.2 vimos resultados que permiten deducir el MCD de a y b cuando a = b,


cuando a = 0 o b = 0 y cuando b divide a a. En la bsqueda ms general del MCD se puede suponer
0 < b < a.
1. Resultado preliminar
Efectuando la divisin euclidiana de a por b, a = bq + r con 0 r < b.
Entonces los divisores comunes de a y b son los divisores comunes de b y r, por lo que se puede
deducir que D(a, b) = D(b, r). En particular si r = 0, D(a, b) = D(b, 0) = D(b).
Demostracin. Primero demostramos que todo divisor c de a y b divide tambin a b y r.
Solo falta probar que l divide a r. Tenemos que r = a bq, luego c divide a a y b, c divide a r
(Captulo 1, subseccin 1.3, propiedad 5) y tambin a b.
Recprocamente, demostramos que todo divisor d de b y r divide tambin a a y b. Slo falta
probar que d divide a a. Como a = bq + r, entonces como d divide a by r entonce d divide a a y
tambin a b.

21

En conclusin los divisores comunes a a y b son los divisores comunes a b y r, de esta forma
D(a, b) = D(b, r).
2. Algoritmo de Euclides (cuando b no divide a a)
Demostracin. As la bsqueda de D(a, b) se reduce a la bsqueda de D(b, r), que es estrictamente un par de nmeros "menor" en el sentido que b < a y r < b. Siendo r diferente de cero
(porque b no divide a a) podemos dividir b por r, obtener un residuo r1 y reemplazar (b, r) por
(r, r1 ) y D(a, b) = D(b, r) = D(r1 , r). Y as sucesivamente hasta que se obtenga un residuo igual a
cero.
Ciertamente que al final encontramos un residuo que sea nulo puesto que los residuos sucesivos
r, r1 , r2 , . . . son enteros positivos que van decreciendo estrictamente. Uno obtiene de esta manera
las igualdades: D(a, b)= D(b, r)= D(r, r1 )= D(r1 , r2 )= . . .= D(rn , 0).
Y tenemos que D(rn , 0) = D(rn ).
Como D(a, b) = D(rn ) y ya que rn es el divisor ms grande de D(rn ), rn es el mayor divisor de
D(a, b)y por lo tanto el MCD de a y b.
Consecuencia practica: Cuando b no divida a a,el MCD de a y b es el ltimo residuo no nulo
que se obtiene en el algoritmo.

2.2.

Un resultado importante

Teorema 1. Todos los divisores comunes a dos enteros positivos a y b tambin son divisores
del MCD g, es decir D(a, b) = D(g).
Demostracin. En efecto, en vista que D(a, b) = D(rn ) y rn es el MCD.
Observacin. El teorema nos dice en particular que todo divisor de a y b es un divisor de su
MCD.

22

Cmo utilizar el algoritmo?


Calcular el MCD de dos nmeros a = 138807, b = 52089 y utilizando el algoritmo de Euclides.
Al dividir a por b:
138807 = 2 52089 + 34629
r

Al dividir b por r:
52089 = 1 34629 + 17460
r

r1

Al dividir r por r1 :
34629 = 1 17460 + 17169
r1

r2

Al dividir r1 por r2 :
17460 = 1 17169 + 291
r2

r3

Al dividir r2 por r3 :
17169 = 59 291 + 0
r3

r4

El ltimo residuo distinto de cero es r3 = 291, por lo que r3 es el MCD de a y b.


Observacin: El hecho de saber el MCD de 138 807 y 52 089 permite conocer todos sus divisores
comunes: los divisores comunes a 291 los cuales son 1, 3, 97 y 291.
Cuando calculamos el MCD "a mano", podemos adoptar el sistema en el cual escribimos las divisiones sucesivas una despus de otra, los cocientes los ponemos en la lnea superior (porque no son
utilizados) y los residuos son puestos en la tercera linea.
138 807
34 629

2
52 089
17 460

1
34 629
17 169

1
17 460
291

1
17 169
0

59
291

Ejercicios resueltos
Ejercicio 2. Utilizando la definicin del MCD
Sean a y b dos enteros positivos, a = 630.
El MCD de a y b es igual a 105 y 600 < b < 1100.
Encontrar b.

23

Solucin
Puesto que b es un mltiplo de 105, escribimos b = 105k con k entero positivo.
La condicin 600 < b < 1100 se traduce por:
600 < 105k < 1100
De donde los valores posibles de k son 6, 7, 8, 9, 10, 11, resta encontrar los valores que satisfagan
las condiciones.
k = 6 da b = 6 105 = 630 pero entonces b = a y 105 no puede ser el MCD.
k = 7 da b = 7 105 = 735. Si calculamos el MCD de a y b encontramos 105.
k = 8 da b = 8 105. a = 6 105 y 105 no es el mximo comn divisor puesto que 2 105
divide tambin a a y b.
k = 9 da b = 9 105 y 3 105divide a a y b por lo que 105 no es el MCD.
k = 10 da b = 10 105 y 2 105 divide a a y bpor lo que 105 no es el MCD.
El nico valor que satisface es por lo tanto es k = 7 donde b = 735.

3.

Teorema de Bezout

Los nmeros tales su MCD es igual a 1 juegan un papel importante. En particular, porque
permiten una caracterizacin prctica del MCD.

3.1.

Definicin

Definicin: Dos nmeros naturales se llaman primos entre s, si su MCD es igual a 1.


Se dice incluso que no tienen otro divisor comn que no sea 1.
Ejemplo: 8 y 15 son primos entre s.
Observacin: La definicin se extiende a nmeros enteros. Dos enteros son primos entre si, cuando
los nicos divisores comunes son 1 y -1.

3.2.

Caracterizacin: El Teorema de Bezout

Teorema 2. Sean a y b dos nmeros naturales no nulos. Decir que: a y b son primos entre s
es equivale a decir que existen dos nmeros enteros u y v, tales que, au + bv = 1.
Demostracin
1. Supongamos que existen dos enteros u y v, tales que au + bv = 1, y probamos que a y b son
primos entre si.
El MCD g de a y b, divide a y b, por ende divide a todo nmero de la forma au + bv con u y
v enteros. Pero por hiptesis existen enteros u y v tales que au + bv = 1, por consiguiente, g
divide 1, de donde g = 1, y a y b son primos entre si.
2. Supongamos que a y b son primos entre si y probemos que 1 se escribe como au + bv. Consideremos el conjunto E de todos los nmeros au + bv con u y v en Z.
24

E contiene a a porque, a = a(1) + b(0), por ende E contiene enteros estrictamente positivos y
entre ellos hay uno mas pequeo que todos los dems. Denotemos por m a dicho entero, as
m = au1 + bv1 con u1 Z y v1 Z. Demostremos que m divide a a y a b; de donde resulta
que m = 1 y que au1 + bv1 = 1.
La divisin de a por m, con m = au1 + bv1 , nos da: a = (au1 + bv1 )q + r, de donde
r = a(1qu1 ) + b(qv1 ) = au + bv con u y v en Z.
Puesto que r est en E y dado que, 0 r < m y siendo m el entero mas pequeo de E
positivo, necesariamente r = 0 y en consecuencia m divide a. De manera anloga se prueba
que m divide a b.
Ejemplos:
8 7 + 11 (5) = 1; 8 y 11 son primos entre si.
Dos naturales consecutivos y mayores a 1 son primos entre si, ya que n 1 + (n
1) (1) = 1.
Ejercicio 3: Probar que dos nmeros son primos entre si.
Si n es un entero natural. Demuestre que a = 2n + 1 y b = 3n + 2 son primos entre si.
Solucin comentada.
Tratemos de encontrar dos enteros u y v, tales Ponemos entonces, u = 3 y v = 2 para que
que para todos n, (2n + 1)u + (3n + 2)v = 1. La el resultado sea +1. Esto da: 3a + 2b = 1 .
idea para eliminar n elegiendo u = 3 y v = 2 Ejercicios: 12 y 13, 21.
nos da resultado 1.
Ejercicio 4: Resultados que es preferible conocer o saber demostrar.
1. Demostrar que si un natural a es primo con dos enteros b y c, tambin es primo con su
producto.
2. Deducir que si a y b son primos entre si, entonces an y bp son primos entre si, para todo
natural n 1, para todo natural p 1.
Solucin comentada.
1. a es primo con cada uno de los nmeros
enteros b y c. Segn el Teorema de Bezout, existen enteros u, v, u, v, tal que,
au + bv = 1 y au + cv = 1. Multiplicando, miembro a miembro, las dos
igualdades se obtiene: a(auu + cuv +
bvu) + (bc)(vv) = 1, que es de la forma
ax+(bc)y = 1 con x, y y enteros. En consecuencia, segn el teorema de Bezout, a
es primo con bc.

As a es primo con b y b2 , y por ende con b3 . Y


por induccin se establece que a es primo con
bp . Como bp es primo con a, tambin es primo con a2 . Luego, bp es primo con a y primo
con a2 , por lo que es primo con a3 , y por recurrencia se concluiye que bp es primo con an
cualquiera que sea n. Ejercicio 25.

2. Como consecuencia del numeral anterior,


con c = b, se obtiene que a primo con
b b = b2 .
Ejemplo 5. Otros resultados interesantes.
Sean a y b dos naturales primos entre si.
25

1. Pruebar que a + b es primo con a y con b.


2. Deducir que a + b es primo con ab, y que a2 + b2 es primo con ab.
Solucin comentada.
Ya que a y b son primos entre si, se puede
escribir au + bv = 1 con u y v en Z. Sumando
un cero conveniente que nos permita introducir
a + b, obtenemos: au + bubu + bv = 1, es
decir, (a + b)u + b(v u) = 1, lo que prueba
que a + b y b son primos entre si. Demuestre
del mismo modo que a + b es primo con a.

26

De lo demostrado anteriormente, a+b es primo


con ab. Se deduce (ver el ejercicio resuelto 4)
que (a + b)2 es primo con ab (para introducir
a2 + b2 ). De donde se deduce que existen u y
v tales que, u(a + b)2 + vab = 1 con u y v en
Z. As que, u(a2 + b2 ) + 2abu + vab = 1 luego:
u(a2 + b2 ) + (2u + v)ab = 1, lo que demuestra
que a2 + b2 y ab son primos entre si.

4.

Caractersticas y Propiedades del MCD

4.1.

Caractersticas y Propiedades del MCD

Teorema 3. Sean a, b y g enteros estrictamente positivos. Las tres proposiciones siguientes son
equivalentes.
(1) g es el MCD de a y de b
a b
(2) g es un divisor de a y b y los enteros positivos y son primos entre s.
g g
(3) g es un divisor de a y b y existen enteros u y v tales que au + bv = g.
Demostracin. 1. Probemos que (1) implica (2). Supongamos que g es el MCD de a y de b.
b
a
= a0 y
= b0 son naturales. Probemos que son
Luego g divide a a y b de donde los nmeros
g
g
primos entre s. Si d es un divisor comn de a0 y b0 , entonces a0 = dp, b0 = dq, donde p y q son
naturales.
Por lo tanto a = dgp y b = dgq, de donde dg sera un divisor comn de a y b. Ahora bin, g es
el mayor de estos divisores as que d = 1, y entonces a0 y b0 son primos entre s.
2. Probemos que (2) implica (3). Supongamos que d es un divisor de a y b y que los enteros
a b
a
b
naturales y son primos entre s. Entonces por el teorema de Bezout podemos escribir u+ v =
d d
g
g
1 con u y v en Z, por tato au + bv = g.
3. Probemos que (3) implica (1). Supongamos que g es un divisor de a y b y que existen dos
enteros u y v tales que au + bv = g.
Sea g 0 el MCD de a y de b. Ya que g divide a a y a b, tambin divide a g 0 (seccin 1.4 de este
captulo).
Sin embargo g 0 divide tambin a au + bv, es decir divide g. Por lo tanto g = g 0 .
Nota: Hemos demostradoque las tres proposiciones son equivalentes entre s ya
que las implicaciones probadas forman un argumento cclico. As, por ejemplo (1)
es equivalente a (2), porque (1) implica (2) y (2) implica (3) y (3) implica (1). Por
lo tanto (1) equivale a (2).

4.2.

Resultado: Propiedad multiplicativa del MCD

Si g es el MCD de dos naturales a y b, entonces para cualquier entero c, c > 0, gc es el MCD de


ac y de bc.
Demostracin. Comog divide a a y b, entonces gc divide a ac y bc. Para demostrar que gc es el
MCD de ac y bc, es suficiente demostrar que, basado en el teorema previo, que gc se puede escribir
de la forma gc = (ac)u + (bc)v. Ahora g = au + bv, con u y v en Z de donde gc = (ac)u + (bc)v.
De la misma forma se demuestra que si c divide a a y b, por lo que divide tambin a g (teorema
g
a b
1), entonces es el MCD de y .
c
c c

Cmo utiliza el teorema 3?


27

A menudo es til traducir la hiptesis "g es el MCD de a y b" por "escribamos a = ga0 y
b = gb0 con a0 y b0 primos entre s" (Iejercicio resuelto 6).
Si se tienen las igualdades a = ca0 ,b = cb0 y se puede demostrar que a0 y b0 son primos entre
s, entonces se podemos afirmar que c es el MCD de a y b (Iejercicio resuelto 7).
A veces puede ser til traducir el prrafo: "g divide a a y b" por "existen u y v enteros tales
que au + bv = g" (Iejercicio resuelto 8).
Ejercicio 6. Utilizando las caractersticas del MCD
Encontrar dos naturales tales que a + b = 112 y MCD(a, b) = 14.
Solucin
El MCD de a y b es 14 entonces a = 14a0 y b = 14b0 con a0 y b0 enteros primos entre s. Reemplazando
en la igualdad a + b = 112 obtenemos 14(a0 + b0 ) = 112 que equivale a a0 + b0 = 8.Como a0 y b0 son
primos entre s, la soluciones son a0 = 1, b0 = 7 o a0 = 3, b0 = 5 o a0 = 5, b0 = 3 o a0 = 7, b0 = 1.
Multiplicando por 14 encontramos que los pares (14; 98), (42; 70), (70; 42), (98; 14). Resta probar
que en efecto son las soluciones de la ecuacin. Compruebe que esta de acuerdo con la solucin
(Iejercicio resueltos 17, 18 y 20).
Ejercicio 7. Utilizando las caractersticas del MCD
Sean p y n dos enteros, p > 1 y n > 1; a = pn y b = p(n 1).
Demostrar que el MCD de a y b es igual a su diferencia.
Solucin.
Notamos que p = a b.
Por otro lado, p es un divisor comn de a y de b y los cocientes de a y b por p son los enteros n y
(n 1), primos entre s.
De acuerdo a el teorema 3, p es el MCD de a y b. (Iejercicio resuelto 21)
Ejercicio 8. Un resultado general
Sea g el MCD de a y b y sea c un entero primo con b.
Demostrar que g es el MCD de ac y de b.
Solucin.
Como g es un divisor de b, divide evidentemente a ac y b.
Traduzcamos las hiptesis "g es el MCD de a y b " luego "existen u y v enteros tales que g = au+bv".
"b y c primos entere s" donde "existen u0 y v 0 enteros tales que 1 = bu0 + cv 0 ". Multiplicando
miembro a miembro ambas igualdades y ordenando para que aparezcan ac y b obtenemos:
(ac)(uv 0 ) + b(auu0 + bvu0 + cvv 0 ) = g
donde g divide a ac y b por lo que, de acuerdo al teorema de Bezout, g es el MCD de ac y b.

5.
5.1.

Aplicaciones
Teorema de Gauss

Teorema. Sean a,b y c enteros estrictamente positivos tales que a divide al producto bc y a es
primo relativo con b. Entonces a divide a c.

28

Demostracin. Como a y b son primos entre s, existen dos enteros u y v tales que au + bv = 1
(teorema de Bezout). Por lo que acu + bcv = c. Como a divide claramente a acu y por hiptesis
divide a bc, entonces divide a bcv. Por lo tanto a divide a la suma acu + bcv, es decir, divide a c.
Nota. El teorema de Gauss se puede enunciar diciendo que si un natural divide a un producto
de dos factores y es primo relativo con uno de ellos, entonces divide al otro.
Corolario. Si un natural n es divisible por dos naturales a y b primos entre s , entonces es
divisible por el producto de ellos.
Demostracin. Por hiptesis, podemos escribir n = ap y n = bq donde p y q son enteros naturales.
Como ap = bq y ya que b divide a ap y es primo relativo a a entonces divide a p. Luego p = bp0 y
p0 es natural, de donde n = abp0 lo cual prueba el corolario.
Este resultado se extiende fcilmente para el caso de varios factores: si n es divisible por mas
de un par de nmeros primos entre s, entonces n es divisible por el producto.
Ejemplos:
Si un nmero es divisible por 3 y 8, entonces es divisible por 24. Por lo que para probar que
un nmero es divisible por 24, solo es necesario que sea divisible por 3 y 8.
Si un nmero es divisible por 3, 5 y 8 entonces es divisible por 3 5 8 = 120, ya que 3, 5 y
8 son primos entre s. Por lo que para probar que un nmero es divisible por 120 es suficiente
probar que es divisible por 3, 5 y 8.

5.2.

Fracciones irreducibles

a
tales que a y b son enteros con b 6= 0. (En lo
b
que sigue consideramos que las fracciones son positivas.)
a
Definicin. Cuando a y b son primos entre s, entonces la fraccin es irreducible.
b
Teorema. Cualquier fraccin es igual a una fraccin irreducible.
c
Demostracin. Consideremos una fraccin . Denotemos por g el MCD de c y d. Luego c = gc0 y
d
c
c0
c0
d = gd0 con c0 y d0 primos entre s. As mediante la simplificacin por g , = 0 con 0 irreducible
d
d
d
puesto que c0 y d0 son primos entre s.
c
a
Cmo explotar el hecho que una fraccin sea igual a una fraccin irreducible ?
d
b
Podemos afirmar que existe un entero positivo k tal que c = ka y d = kb tal que k es el MCD de c
y d.
c
a
a
En efecto, escribamos = , bc = ad y cmo es irreducible, a y b son primos entre s.
d
b
b
Tenemos que a divide a bc y a es primo con b, donde a divide a c (teorema de Gauss). Por lo que
c = ka. Sustituyendo en la igualdad bc = ad , obtenemos bka = ad donde d = kb.
Como k es un divisor comn de c y d y los cocientes a y b son primos entre s.
Por lo tanto, k es el MCD de c y de d (teorema 3).
Llamamos fracciones a los nmeros de la forma

Ejercicios resueltos
29

Ejercicio 9.
Cuatro enteros estrictamente positivos a, b, c, d forman, en ese orden, un sucesin aritmtica
cuya diferencia de la progresin es el nmero primo con a. Encuentre estos nmeros sabiendo que
10a2 = d b.
Solucin.
Llamamemos r a la diferencia de la progresin. Entonces: b = a + r, c = a + 2r, d = a + 3r. La
relacin propuesta por el ejercicio establece que 10a2 = 2r, lo que equivale a 5a2 = r. Como r es
primo con a, tambin lo es con a2 . Como r divide a 5a2 , el teorema de Gauss permite concluir que
r divide a 5. Entonces r = 1 o r = 5. La respuesta r = 1 resulta en no soluciones por lo que r = 5
. De ello se deduce que a2 = 1 por lo que a = 1 dado que la sucesin es de nmeros positivos. Los
nmero buscados son entonces a = 1, b = 6, c = 11, d = 16.
Observacin: Se utiliz la propiedad "Si r es primo con a entonces es primo con a2 ".
Ejercicio 10.
Pruebe que para todo entero n, el nmero A = n(5n2 + 1) es divisible por 6.
Solucin.
Gracias al "corolario", es suficiente probar que el nmero es divisible por 2 y por 3.
Probemos que A es divisible por 3.
El nmero n se puede escribir en una de las forma n = 3k, n = 3k + 1 o n = 3k + 2.
Si n = 3k, A = 3 k 5(3k)2 + 1 es mltiplo de 3.
Si n = 3k+1, entonces A = (3k+1)(45k 2 +30k+6), por lo que A = 3(3k+1)(15k 2 +10k+2) = 3K.
Si n = 3k+2, entonces A = (3k+2)(45k 2 +60k+21), por lo que A = 3(3k+2)(15k 2 +20k+7) = 3K 0 .
Para probar que A es divisible por 2, uno procede en la misma forma estudiando los casos:
n es par o n es impar
Observacin:
La utilizacin de las congruencias (captulo 3) vuelve la redaccin de las solucin ms fcil (lo que
peermite juzgar la eficacia del mtodo)
Iejercicio resuelto 28 y 29

6.

Mnimo Comn Mltiplo

6.1.

Definicin

Dos enteros a y b estrictamente positivos tienen al menos un comn mltiplo, que es el producto
ab. Por lo que como el conjunto de mltiplos comunes no es vaco, de entre todos estos nmeros
existe uno que es el menor de todos.
Notacin.
El mnimo comn mltiplo positivo de a y b es denotado MCM.

6.2.

Resultados fundamentales

Teorema. Si a y b son enteros estrictamente positivos con g su MCD y m su MCM entonces:


ab
o mg = ab;
1. m =
g
30

2. Todos los mltiplos comunes de a y b son un mltiplos de m


.
Demostracin. Sea g el MCD de a y b donde a = ga0 y b = gb0 con a0 y b0 primos entre s. Sea
M un mltiplo comn de a y b. Por lo que resulta que M = ap y M = bq donde p y q son enteros.
Como M = ga0 p = gb0 q entonces, a0 p = b0 q [1]. La igualdad [1] prueba que a0 divide a b0 q. Como a0
y b0 son primos entre s, de acuerdo con el teorema de Gauss, a0 divide a q. Por lo que resulta que
q = ka0 y teniendo en cuenta [1] y p = kb0 . Luego todo mltiplo comn a a y a b se puede escribir
en la forma M = kga0 b0 . Recprocamente, demostremos que todo nmero que se puede escribir en
la forma M = kga0 b0 es un mltiplo comn de a y b. M = k(ga0 )b0 = kb0 a. De la misma forma
M = k(gb0 )a0 = ka0 b.
Por lo tanto los mltiplos comunes a a y b son mltiplos de ga0 b0 . El mnimo de todos estos es
exactamente el MCM m = ga0 b0 . As que mg = ga0 gb0 = ab y cualquier mltiplo de a y b es mltiplo
de m.

6.3.

Caracterizaciones

Teorema. Sean a y b enteros positivos. Decir que "m es el MCM de a y b" es equivalente a
m m
y
son primos entre s".
decir que "m es un mltiplo de a y b y que los enteros naturales
a
b
Demostracin.
1. Sea m el MCM de a y b. Vimos que m = ga0 b0 = ab0 = a0 b. Los cocientes respectivos de m
por a y b no son otros que los nmeros b0 y a0 que son primos entre s.
M M
2. Recprocamente sea M un mltiplo comn de a y b tal que los naturales
y
son primos
a
b0
0 0
entre s. Hemos visto que M = kga b luego los cocientes respectivos por a y b son kb y ka0 .
Como k es un divisor comn de estos dos nmeros y como por hiptesis que son primos entre
s, k = 1. As M = ga0 b0 que es el MCM de a y b.
Cmo utilizar el hecho que m sea el MCM de a y de b?
Sea m y g el MCM y el MCD de a y b respectivamente.
La formula mg = ab es fcil de memorizar pero en la prctica es til escribir m = ga0 b0 donde los
enteros a0 y b0 son los cocientes respectivos de a y b por su MCD. El hecho que a0 y b0 son primos
entre s puede ser til en algunas ocasiones (Iejercicio resuelto 11).
Cmo conocer el MCM de ac y bc a partir del MCM de a y b ?
La siguiente propiedad es llamada "propiedad multiplicativa de del MCM". Si m es el MCM
de dos enteros positivos a y b, entonces para cualquier numero natural c diferente de cero, mc es
el MCM de ac y bc.
En efecto, sea g es el MCD de a y b entonces sabemos que

ac bc
ab
=
c = mc
gc
g
ab
es el MCM de a y b.
g
El MCD de ac y bc es gc.
El MCM de ac y bc es mc.
Ahora bien m =

Ejercicios resueltos
31

Ejercicio 11. Encontrar dos nmeros sabiendo su relacin entre su MCD y MCM.
Encontrar dos enteros a y b tales que la diferencia entre su MCM y MCD es igual a 187.
Solucin.
El enunciado del problema no pide hallar todos los enteros demanda nicamente dos de ellos.
Podemos restringirnos a enteros positivos.
Llamemos a y b los dos nmeros naturales buscados.
Sea m el MCM y g el MCD de a y de b.
As:
a = ga0 , b = gb0
con a0 y b0 primos entre s:
m=

ab
= ga0 b0 .
g

Con g(a0 b0 1) = 187.


As g es un divisor de 187 al igual que a0 b0 1.
Los divisores de 187 son 1, 11, 17 y 187; de donde se obtienen los posibles resultados:
1
188

g
a0 b0

11
18

17
12

187
2

Escogiendo g = 1 a0 b0 = 188 y a0 y b0 primos entre s solucionara la pregunta.


Si escogemos a0 = 4 y b0 = 47, 4 y 47 son primos entre s. De esta forma a = 4 y b = 47, que
cumplen con los requerimientos del problema.

Ideas y Reflexiones
IIdeas para recordar
Divisin y divisin euclidiana
En la divisin euclidiana de a por b , el resto es positivo o nulo y estrictamente inferior
a |b|.
Cuando se divide todos los entero por un entero b fijo, slo existen un nmero |b| de restos
posibles:
0, 1, 2, . . . |b| 1. Ejemplo:
Si b = 4, todo entero se puede escribir de una de las formas: 4q o 4q + 1 o 4q + 2 o 4q + 3.
La divisibilidad es transitiva: si a divide a b y b divide a c entonces a divide a c.
Si c divide a a, entonces c divide a todos los nmeros au, y si c divide a a y b , entonces c
divide a todos los nmeros au + bv.

MCD y MCM
Los divisores comunes de a y b son los divisores de su MCD.

32

Decir que dos nmeros son primos entre s es decir que su MCD es igual a 1, o que 1 es su
nico divisor comn.
El MCD g de a y b es tambin el divisor comn de a y b tal que

a b
y son primos entre s.
g g

El MCD g de a y b tambin es un divisor comn a a y b escrito en la forma au + bv con u y


v en Z.
a
Saber si la fraccin es irreducible es tambin saber si a y b son primos entre s.
b
El MCM de a y b es fcil de encontrar a partir de su MCD puesto que:
MCM MCD = ab.
u u
El MCM de a y b, u, es tambin el mltiplo comn de a y b tal que y son primos entre
a
b
s.

ISugerencias
Piense las relaciones de divisibilidad en trminos de igualdades.
Ejemplo: Conocido que b divide a a, podemos escribir a = bq con q entero (e incluso q > 0 si
a > 0 y b > 0).
Recprocamente, trate de interpretar las igualdades en trminos de divisores (o de mltiplos).
Ejemplos:
1. Dos enteros x y y tales que 3x + 27 = y, concluimos que 3 divide divide a y.
2. Dos enteros a, b, c son tales que 4a + 5b = c. Por lo que deducimos que si d divide a a y
b entonces d divide a c.
Esencial: Recuerde que si d divide a a y b, entonces d divide a cualquier nmero de la forma
au + bv, y divide a cualquier suma o diferencia de nmeros de esta forma.
Ejemplo: Si d divide a ab y a + b entonces d divide a a2 . En efecto, d divide tambin a
a(a + b) = a2 + ab y ab por lo que divide a a2 + ab ab. De igual manera divide a b2 .
Una manera de demostrar que a divide a b es escribir la divisin euclidiana de b por a y a
continuacin demostrar que el residuo es nulo.
Para demostrar que a y b son primos entre s sin calcular su MCD, se puede suponer que d
divide a a y b y probar que d = 1 a travs de una hiptesis propuesta.
Para demostrar que n divide a un entero a , se puede escribir n = n1 n2 con n1 y n2 primos
entre s, y enseguida probar que n1 y n2 dividen a a.

IErrores a evitar
Una igualdad a = bq no significa que b divide a a sin estar seguro que q es un entero.
La ecuacin a = bq + r no siempre refleja una divisin euclidiana. En efecto, es indispensable
aadir la condicin 0 r < b.
33

Algoritmo para calcular los coeficientes de las relaciones de


Bezout
Sean a y b dos enteros naturales, a > b > 0, y g su MCD. Entonces existen u y v enteros tales
que au + bv = g. El propsito de este taller es hallar un algoritmo para calcular u y v.

1. Determinar u y v a travs de un ejemplo numrico


Mtodo.
Podemos utilizar el algoritmo de Euclides en calcular restos sucesivos en funcin de a y b.
Sabemos que el ltimo resto no nulo es el MCD. En el desarrollo de estos clculos llegamos a una
etapa en la escritura de la forma au + bv que es igual al MCD.
Tomemos a = 47 y b = 35
47 = 35 1 + 12
o a = b + 12
as que 12 = a b
35 = 12 2 + 11
b = (a b) 2 + 11
11 = 2a + 3b
12 = 11 1 + 1
a b = (2a + 3b) 1 + 1 1 = 3a 4b
11 = 11 1 + 0
Hemos obtenido el MCD como una combinacin linear de a y b.

2. Generalizaciones
1. Anlisis de la situacin
En la presentacin del algoritmo de Euclides (En Captulo 1.3) vimos el clculo de restos sucesivos ri hasta que el resto se convierte en nulo. Para homogenizar la notacin, es conveniente atribuir
r0 = a y r1 = b.
A continuacin, la relacin ri = ri+1 qi+1 + ri+1 [1] permite obtener el resto ri+2 a partir de
sus dos precedentes ri+1 y ri .
En consecuencia (ri ) es una secuencia definida por la relacin de recurrencia [1] donde los dos
primeros trminos r0 = a y r1 = b.
La situacin se puede abreviar de la siguiente forma:
divisin 1
divisin 2
...
divisin i + 1
...
divisin n 1
divisin n

r0 = a; r1 = b
r0 = r1 q1 + r2 (o a = bq1 + r2 )
r1 = r2 q2 + r3
ri = ri+1 qi+1 + ri+2

[1]

rn2 = rn1 qn1 + rn


rn1 = rn qn + rn+1 con rn+1 = 0

Solucin de la ecuacin au + bv = c
1. Presentacin del problema
Sean a,b, y c nmeros enteros, la ecuacin ax + by = c con x y y nmeros desconocidos en Z se
llama ecuacin diofntica.
34

Las ecuaciones diofnticas son ecuaciones algbricas con varios trminos desconocido cuyos trminos desconocidos son enteros o racionales donde se buscan las soluciones enteras.
A menudo son ecuaciones delicadas para resolver. Es fcil encontrar las soluciones reales de
3x + 4y = 0 que las soluciones enteras.

2. Estudio de un caso particular


En el presente apartado, nos centraremos en la ecuacin au + bv = g cuando g es el MCD de a
y b.
1. Buscar la solucin particular (u0 , v0 ) de la ecuacin au + bv = g.
Notamos (u, v) una solucin alternativa a la ecuacin y denotamos por a0 =

b
a
y b0 = .
g
g

a) Justifiquemos la igualdad a0 (uu0 )+b0 (uu0 ) = 0, a continuacin utilizamos el teorema


de Gauss para demostrar que existe un entero relativo k tal que u = u0 kb0 y v = v0 +ka0 .
b) Recprocamente probamos que si u = u0 kb0 y v = v0 + ka0 con k entero, entonces
au + bv = g.
2. Aplicacin
a) El MCD de 114 y 30 es 6.
Encontrar todas las parejas (u, v) enteras tales que 114u + 30v = 6.
b) Interprete grficamente los resultados.

3. Caso general
Supongamos que c es un entero cualquiera y a y b enteros donde a > b > 0.
1. a) Pruebe que si c no es un mltiplo del MCD de a y b entonces la ecuacin ax + by = c no
tiene soluciones.
b) Supongamos que c es un mltiplo del MCD de a y b (c = c0 g). Resolver la ecuacin
ax + by = c.
2. Aplicacin.
a) En un plano cartesiano, dibujar la lnea recta d de la ecuacin 47x + 47y = 1.
Buscar los puntos en d donde las coordenadas son enteras.
b) La misma pregunta para la lnea recta determinada por la ecuacin 33810x+4116y = 588.

Ejercicios
Para resolver los ejercicios de esta parte, basta con aplicar directamente los teoremas presentados
en esta seccin. El objetivo es doble: comprender mejor el curso y reconocer, en situaciones simples,
en las condiciones de aplicacin de un teorema.

35

Mximo Comn Divisor.


1. En casa caso, encuentre el conjunto de divisores D(a) y D(b) de los naturales a y b. En cada
caso, determine el Mximo Comn Divisor.
a) a = 24; b = 18.
b) a = 150; b = 240.
c) a = 60; b = 84.
d ) a = 150; b = 77.
2. Si a y b son dos nmeros naturales, a 2b. Demuestre que el conjunto de divisores comunes
de a y b es el mismo que el conjunto de los divisores comunes de b y a 2b.
3. Utilice el algoritmo de Euclides para encontrar el MCD de los nmeros siguientes:
a) 360 y 2100.
b) 468 y 312.
c) 400 y 840.
4. Utilice el algoritmo de Euclides para encontrar el MCD de los nmeros siguientes y en cada
caso, determine el conjunto de divisores comunes de estos nmeros.
a) 144 y 840.
b) 202 y 138.
c) 147 y 490.
5. Encuentre el conjunto de todos los divisores comunes de a y b.
a) a = 300; b = 350.
b) a = 168; b = 2160.
c) a = 308; b = 364.
6. Si se divide 4 294 y 3 521 por un mismo entero positivo, y se obtiene respectivamente 10 y
11 como residuo. Cul es este entero? Sugerencia: Todo divisor comn de dos enteros es un
divisor de su MCD.
7. Sea b es un natural tal que 260 < b < 300. El MCD de a = 600 y de b es 12. Encuentre b.

Teorema De Bezout.
8. Son los nmeros a y b son primos entre si?
a) a = 42; b = 65.
b) a = 147; b = 350.
c) a = 442; b = 693.
36

9. Encuentre todas las parejas de nmeros naturales primos entre si y cuya suma sea 12.
10. Sean a y b dos enteros, a = 18. Encuentre todos los valores de b sabiendo que b es primo con
a y 20 < b < 30.
11. Sea n un nmero natural cualquiera; a = 3n + 4 y b = 2n + 3. Pruebe que si d divide a a y b,
entonces d divide a 1. Qu se puede deducir para a y b?
12. Sea n es un entero natural; a = 2n + 1; b = 3n + 2. Encuentre una relacin entre a y b,
independiente de n. Deduzca que a y b son primos entre s.
13. Sea n es un entero natural; a = 7n + 4; b = 5n + 3. Encuentre una relacin entre a y b
independiente de n. Deduzca que a y b son primos entre si.
14. Utilice el algoritmo de Euclides para encontrar una pareja (x, y) de enteros tales que 89x +
41y = 1.
15. Utilice el algoritmo de Euclides para encontrar una pareja (x, y) de enteros tales que 59x +
27y = 1.

Caractersticas y Propiedades del MCD.


16. Explique el porqu en cada uno de los casos siguientes, se puede encontrar muy rpidamente
el mximo comn divisor de a y de b.
a) a = 5 12; b = 11 12.
b) a = 3 15; b = 8 15.
c) a = 22 26; b = 15 26.
d ) a = 12 20; b = 35 20.
e) a = 32 12; b = 75 12.
f ) a = 100 24; b = 63 24.
17. Encuentre dos nmeros naturales a y b cuyo MCD es 4, tal que a + b = 48.
18. Encuentre dos nmeros enteros naturales a y b cuyo MCD es 8, tal que a + b = 144.
19. Encontrar las soluciones del sistema:


xy = 7875
MCD(x, y) = 15

Si (u, v) denota una solucin del sistema y g el MCD de u y de v; u0 y v 0 son los enteros
v
u
u0 = y v 0 = .
g
g
a) Qu se puede decir de u0 y v 0 ?
b) Calcule u0 v 0 y deduzca los valores posibles de (u0 , v 0 ). Deduzca los valores posibles de
(u, v). Verifique que estas son las soluciones del sistema dado.
37

20. Sean a y b dos nmeros naturales. Encuentre a y b sabiendo que ab = 1734 y que el MCD de
a y de b es 17.
21. Sea n un entero natural; a = 2n2 y b = n(2n + 1). Justifique que 2n y 2n + 1 son primos entre
si. Deducir que n es el MCD de a y b.

Aplicaciones.
22. En cada caso, encuentre todas las parejas de nmeros naturales (x, y) que cumplan las condiciones:
a) 6y = 11x y 0 x < 25.
b) 7y = 3x y 0 x < 15.
23. Encuentre todas las parejas de nmeros naturales (x, y) tales que: 4(x + 3) = 3y.
24. Encuentre todas las parejas de nmeros naturales (x, y) tales que: 3(x 2) = 5(y + 3).
25. Sean a y b dos nmeros naturales primos entre si, tales que a2 + a = 7b3 .
a) Demuestre que a y b3 son primos entre si.
b) Demuestre que a divide a 7. Encuentre los valores de a y b verificando las hiptesis del
texto.
26. Pruebe que la fraccin

n
, n N , es irreducible.
2n + 1

27. El objetivo de este ejercicio es el de encontrar aquellos valores del natural n, n > 4, tal que
n + 17
sea un entero.
la fraccin
n4
a) Demuestre que decir que n 4 divide a n + 17 es equivalente a decir que n 4 divide
a 21.
b) Determine todos los valores de n para que la fraccin dada sea un entero.
28. Sea n es un natural y a = n(n + 1)(n + 5). Pruebe que a es divisible por 6.
29. Sea n es un natural y a = 5(n2 + n)2 . Pruebe que a es divisible por 20.
30. Sea n es un natural, n > 2.
a) Demuestre que el MCD de n + 13 y de n 2 es igual a MCD de n 2 y de 15.
n + 13
sea irreducible?
b) Cules son los valores de n para que la fraccin
n2

38

Mnimo Comn Mltiplo.


31. En cada uno de los casos, encuentre el MCM de los nmeros a y b dados:
a) a = 24; b = 56.
b) a = 180; b = 450.
c) a = 308; b = 4004.
d ) a = 120; b = 300.
e) a = 72 y b = 108.
f ) a = 175 y b = 490.
32. En cada uno de los casos, encuentre el MCD de los nmeros a y b dados y deduzca su MCM:
a) a = 24; b = 56.
b) a = 300; b = 750.
c) a = 1386; b = 546.
d ) a = 30; b = 36.
e) a = 260 y b = 400.
f ) a = 123 y b = 82.
33. Cul es el mas pequeo natural que dividido por 140 y por 252 da 40 como residuo?
34. Resolver para x y y el sistema:

xy = 360
MCM(x, y) = 60

Se denota por (u, v) una solucin del sistema y calcule el MCD g de u y de v.


1. Si u = gu0 y v = gv 0 , qu se puede decir de u0 y v 0 ?
2. Calcule el producto u0 v 0 . Cules son los valores posibles de (u0 , v 0 )?. Deduzca los valores
posibles de (u, v). Verifique que estas son las soluciones del sistema dado.

PARA APRENDER A INVESTIGAR


Los ejercicios en esta parte son de dificultad media en su mayor parte, no se resuelven los
ejercicios. La investigacin quiere mostrar cmo se puede organizar una bsqueda y seguir un
razonamiento.
35. Saber si una fraccin es irreducible. Objetivo: Encontrar los valores del natural n para
n3 + n
es irreducible.
saber si la fraccin
2n + 1

39

a
a) Comprenda el problema. Por definicin, una fraccin
es irreducible cuando a y
b
b son primos entre si. El problema es, pues, encontrar los valores de n para los cuales
2n + 1 y n3 + n son primos entre si. Por tanto, vamos a buscar los divisores comunes
posibles de 2n + 1 y n3 + n.
1) En general, se denota por d a un divisor comn y se utiliza cuantas veces sea necesario
el hecho que si d divide a y b, d divide al nmero au + bv. El problema esta en elegir
bien u y v con el fin de obtener informacin sobre d.
2) En general, tambin en la bsqueda de los divisores de un nmero A, es til si es
posible, factorar A, ya que si A = BC, el conocimiento de los divisores de nmeros
B y C puede conducir al conocimiento de los divisores de A.
Aqu: n3 + n = n(n2 + 1). Denotemos por d, un factor comn de los nmeros 2n + 1 y
n(n2 + 1) y veamos si d puede tener un divisor comn con n, y tambin con n2 + 1.
1) Para elllo, denote por d0 a un divisor comn de d y n. Teniendo en cuenta que si d0
divide a 2n + 1, pruebe que d0 = 1.
2) Qu decir entonces de d y n? Por qu puede decir que d es un divisor comn de
n2 + 1 y 2n + 1?
3) Pruebe que d divide a n(n2) y justifique que d divide a n2. Deduzca d = 1 o d = 5.
b) Hemos demostrado que si d es un divisor comn a 2n + 1 y de n3 + n, entonces d = 1
o d = 5. El caso d = 1 corresponde a una fraccin irreducible. El caso d = 5 es la nica
posibilidad dado que la fraccin es reducible. Supongamos entonces d = 5 y veamos si
podemos sacar conclusiones sobre n.
1) Observando los resultados de las preguntas anteriores, justifique que si d = 5 entonces, n2 = 5q, luego n = 5q + 2 con q entero.
2) Pruebe recprocamente que si n = 5q + 2, entonces, la fraccin es reducible.
3) Concluya.
c) Redacte una solucin.

Divisores Comunes. Mximo Comn Divisor. Nmeros Primos entre


s.
36. Demuestre que para todo entero k, 2k + 1 y 9k + 4 son primos entre si.
37. Sea n es un nmero natural. Encuentre el MCD de n + 1 y n(2n + 1)
38. Para todo nmero natural n, mayor o igual a 5, se consideran los nmeros: a = n3 n2 12n y
b = 2n2 7n4.
a) Demuestre, despus de la factorizar, que a y b son naturales divisibles por n4.
b) Sea a = 2n + 1 y b = n + 3. Denote con d al MCD de a y b.
1) Encuentre una relacin entre a y b independiente de n.
2) Demuestre que d es un divisor de 5.
3) Demuestre que los nmeros a y b son mltiplos de 5, s y solamente s n2 es mltiplo
de 5.
40

c) Demuestre que 2n + 1 y n son primos entre s.


d ) Determinar los valores de n y en funcin de n, el MCD de a y b . Verifique los resultados
obtenidos en los casos particulares n = 11 y n = 12.
39. Sean a y b son dos enteros; A = 11a + 2b y B = 18a + 5b.
a) Demuestre que si uno de dos nmeros A o B es divisible por 19, lo mismo ocurre para
el otro.
b) Si a y b son primos entre si, A y B no pueden tener otros divisores comunes que no sean
1 y 19.
40. Sea a es un entero estrictamente positivo. Si m = 20a + 357 y n = 15a + 187, y g el MCD de
m y de n. Demuestre los cuatro enunciados siguientes:
a) g divide a 323.
b) g es mltiplo de 17 equivale a que a es un mltiplo de 17.
c) "g es un mltiplo de 19 equivale a que" existe un entero k tal que, a = 19k + 4.
d ) 289 es el menor entero a tal que g = 323.
41. Sean a y b dos enteros estrictamente positivos. Sean m = 15a + 4b y n = 11a + 2b. Demuestre
los dos enunciados siguientes:
a) m es un mltiplo de 7, lo que equivale a n es un mltiplo de 7.
b) Cuando a y b son primos entre si, el MCD de m y de n es un divisor de 14.

Aplicaciones del Teorema de Gauss


42. Si n esta un los entero pruebe que n(2n + 1)(7n + 1) es divisible por 6.
43. Sea n es un nmero natural. Se detone a = n(n2 + 5).
a) Demuestre que a es divisible por 2.
b) Explique porqu todo nmero natural n puede ser escrito bajo la forma 3k o 3k + 1 o
3k + 2, siendo k un nmero natural. Verifique que para cualquier n, a es divisible por 3.
c) Qu se puede deducir de las afirmaciones precedentes?
44. Sea n un entero. Pruebe que:
a) n(n + 1)(n + 2)(n + 3) es divisible por 24;
b) n(n + 1)(n + 2)(n + 3)(n + 4) es divisible por 120.
45. Sea n un nmero natural. Pruebe que (n2 1)(n2 )(n2 + 1) es divisible por 60.
46. Sean n y p enteros estrictamente positivos.
a) Demuestre que n(n4 1) es divisible por 30.
41

b) Demuestre que la ltima cifra en la escritura decimal de np y np+4 es la misma cifra. [2]
equivale a np+4 np es divisible por 10.
47. Sean x y y son enteros y (E) es la ecuacin 2x5y = 1.
a) D una solucin particular (x0 , y0 ) de esta ecuacin.
b) Verifique que (E) equivale a 2(xx0 ) = 5(yy0 )
c) Encuentre la solucin general de esta ecuacin.
48. Sean x y y enteros y (E) es la ecuacin: 3x + 4y = 1
a) D una solucin particular (x0 , y0 ) de esta ecuacin.
b) Verifique que (E) equivale a 3(xx0 ) = 4(y0 y)
c) Encuentre la solucin general de esta ecuacin.
49.
a) Verifique que, sea cual sea el valor del natural n, n2 + n y 2n + 1 son primos entre si.
b) Sea n un natural. Demuestre que si n es solucin de la ecuacin 21(n2 +n) = 4[(2n+1)3
40(2n + 1)], entonces 2n + 1 un divisor de 21. Complete la resolucin de esta ecuacin
en N.
50.
a) Aplique el algoritmo de Euclides a los dos enteros a = 41 y b = 27. Deduzca una solucin
particular en de la ecuacin 41x27y = 1.
b) Deduzca de la pregunta anterior una solucin particular en de la ecuacin 41x27y = 5.
c) Resuelva la ecuacin 41x27y = 5.
d ) Pruebe que existen al menos dos enteros relativos k y n tal que 13k23n = 1. Determine
con la ayuda del algoritmo de Euclides dos de estos enteros.
e) Resuelva en la ecuacin 156x + 276y = 24.

MCD, MCM
51. El MCM de dos nmeros es 216. Uno de los nmeros es 72. Cul es el otro?


xy = 1512
xy = 300
52. Resuelva los siguiemtes sistemas:
y
.
MCM(x, y) = 252
MCM(x, y) = 60
1. Encuentre todos los divisores (naturales) de 108.
2. Encuentre todas las parejas (x, y) de enteros naturales donde el MCD d y el MCM m son
tales que: m3d = 108 y 10 < d < 15.
53. Determine todas las parejas (a, b) de nmeros naturales donde el MCM m y el MCD d verifican
la relacin 8m = 105d + 30.
42

54. Resuelva la ecuacin: MCD(x, y) 9MCM(x, y) = 13.


55. Sean a y b son enteros tales que a > b > 0, g es su MCD y m su MCM.
a) Determine g y m cuando a = n(2n 1) y b = (n1)(2n1), con n es entero positivo.
b) Sea p =
(1)

a
g

y q = gb . Exprese en funcin de p y q los nmeros a y b tales que m(a+b) = abg

c) Entre los nmeros a y b que se relacionan por (1), encuentre a aquellos que satisfacen
g = ab (2).
d ) Demuestre que los nmeros enteros a y b que satisfacen a la vez a (1) y (2), son tales que:
(ab)2 = a + b (3) Los nmeros enteros a y b que satisfacen a (3) tambin satisfacen
ellos las relaciones (1) y (2)?
e) Denote por r al resultado de la divisin de a por b y se supone que a y b satisfacen a la
relacin (3). Calcule a y b en funcin de r, para r 6= 0. Aplquelo cuando r = 11.

Fracciones
56. Sea n es un entero cualquiera; a = n3 2n + 5 y b = n + 1.
a) Compruebe que para todo entero n: a = (n2 n 1)b + 6.
b) Demuestre que el MCD(a, b) = MCD(b, 6).
c) Para cada valor de n se tiene que el MCD(a,b) = 3?
a
d ) Determine n para que el nmero sea un entero.
b
57. Sea n es un entero cualquiera. Suponga que A = n1 y B = n2 3n + 6.
a) Demuestre que el MCD de A y B es igual al MCD de A y 4.
b) Determine de acuerdo a los valores del entero n, el MCD de A y B.
c) Para cuales valores del entero n, n 6= 1,

n2 3n + 6
es un entero?
n1

58. Pruebe que si la suma de dos fracciones irreducibles es igual a un entero, entonces los denominadores son iguales.
59. n es un entero, n > 1; suponga a =

n2 + 1
.
n(n2 + 1)

a) Pruebe que el conjunto de divisores comunes del numerador y el denominador de a es el


conjunto de divisores comunes de n2 1 y 2.
b) Deduzca que si n es par, entonces la fraccin es irreducible, y que si n es impar entonces
el MCD del numerador y del denominador es igual a 2.

43

Parte III

Las Congruencias.
Actividad 1. El lenguaje de las Congruencias.
Introduccin a las congruencias
Desde la antigedad, se sabe que ciertos problemas se resuelven analizando los residuos de la
divisin euclidiana. Uno de los problemas ms frecuentes es el clculo de la hora:
Example 1. Si en este momento son las 8:00AM, qu hora ser dentro de 97 horas?
Para resolver este problema, se observa que cada 24 horas ser nuevamente las 8:00AM, as, lo
importante es descomponer 97 en tantos bloques de 24 horas como se pueda y analizar qu sucede
con las horas sobrantes; dado que 97 = 24 4 + 1, cuando se hayan cumplido 96 = 24 4 horas,
ser nuevamente las 8:00AM, por lo que luego de 97 horas ser las 9:00AM.
Es importante notar que problemas como el anterior versan sobre la Divisin Euclidiana, pero
el cociente no es lo realmente importante sino que el residuo y el divisor. En el ejemplo anterior
interesa analizar la posicin que ocupa un nmero a partir de un mltiplo de 24 y no cuntas das
han transcurrido; por comodidad y siempre que sea posible, se prefiere trabajar con el mltiplo que
sea lo ms cercano.
Example 2. Un grupo de 25 estudiantes de matemtica ser seccionado en 6 grupos, d un algoritmo para hacer esto.
Una forma de seccionar o particionar el grupo de estudiantes es la siguiente: se colocan en
una fila y cuentan: 1, 2, 3, 4, 5, 6, 1, 2, 3, 4, 5, 6, 1, 2, . . . desde el primer estudiante de la fila hasta el
ltimo; luego, los estudiantes que dijeron 1 conforman lo que se llamar el Equipo 1, los estudiantes
que dijeron 2 el Equipo 2, etc.1
Remark. Observe que si los estudiantes hubiesen estado numerados previamente del 1 al 25 (en el
orden de la fila), cada estudiante puede conocer a qu grupo pertenecer sin necesidad de esperar
a decir su nmero: por ejemplo, el estudiante 17 sabe que aquellos que su correlativo es mltiplo
de 6 pertenecen a la Clase 6, y por tanto el nmero 17 corresponden a la Clase 5 dado que
se encuentra una posicin antes de un mltiplo de 6 (el 18). As, lo importante ac es conformar
bloques de 6 nmeros y analizar la posicin relativa en alguno de esos bloques. A la longitud de los
bloques se le llamar el Mdulo, y fijado un mdulo se generan muchas clases de nmeros, llamadas
las Congruencias.
Example 3. El conjunto de los nmeros enteros Z puede ser particionado2 de muchas maneras:
Pares e impares: Esto es, los nmeros de la forma 2k llamados pares, y los nmeros de la
forma 2k + 1 llamados impares(con k Z). Esto es segn el ejemplo anterior, los nmeros de
la Clase 0 mdulo 2, y los nmeros de la Clase 1 mdulo 2.
1 Los

matemticos prefieren utilizar el trmino Clase en lugar de Equipo.


un Conjunto significa dividirlo en Clases, las cuales deben de ser mutuamente excluyentes y su unin
conforman todo el conjunto.
2 Particionar

44

Los nmeros de la forma 3k llamados mltiplos de 3, los nmeros de la forma 3k + 1 y los


nmeros de la forma 3k + 2 (con k Z). De nuevo, segn la el ejemplo anterior, esto es los
nmeros de la Clase 0 mdulo 3, los nmeros de la Clase 1 mdulo 3 y los nmeros de la Clase
2 mdulo 3.
Esta claro que esto se puede generalizar para cualquier m = 1, 2, 3, . . .: Los nmeros de la
forma mk llamados mltiplos de m, los nmeros de la forma mk + 1, los nmeros de la forma
mk + 2, . . ., y los nmeros de la forma mk + (m 1) (con k Z). Es decir, los nmeros de la
Clase 0 mdulo m, los nmeros de la Clase 1 mdulo m, . . ., y los nmeros de la Clase m 1
mdulo m.
Remark 4. Tomando un mdulo m arbitrario (m N) es posible particionar a Z en m clases, y
cada nmero entero pertenece a una y slo a una de estas clases mdulo m.
Remark 5. Dado un mdulo m, si se toman dos nmeros a y b de la misma clase, digamos por
ejemplo de la clase r, entonces existen k1 , k2 Z tales que a = mk1 + r y b = mk2 + r, y se cumple
que
a b = (mk1 + r) (mk2 + r) = m(k1 k2 )
La diferencia es un mltiplo de m. Esta observacin es la idea central de toda esta seccin, as, se
dice que dos nmeros son congruentes mdulo m si su diferencia es un mltiplo de m. Entonces,
todos los nmeros de una misma clase (prefijado el mdulo) son congruentes entre s.3
Remark 6. Las Congruencias y la Divisin Euclidiana estn vinculadas, sin embargo, el lenguaje de
las Congruencias es ms flexible y verstil; si se quiere partir el conjunto Z en tres clases (mdulo
3), si lo hacemos tomando de referencia los residuos, stos slo pueden tomar los valores r = 0, r = 1
r = 2 dado que 0 r < 3, formando las clases 0, 1 2 respectivamente; sin embargo, Z tambin
puede ser partido en las clases 3k, 3k + 1, 3k 1, llamadas clases 0, 1, -1, respectivamente. Esto no
es contradictorio en absoluto, porque el conjunto de nmeros de la forma 3k + 2 y el conjunto de
nmeros de la forma 3k 1 es el mismo; as, la clase 2 y la clase -1 coinciden en mdulo 3.

Las congruencias mdulo 4


Claim 7. Un nmero N de la forma 4n + 3 no puede ser escrito como suma de dos cuadrados
perfectos. Veamos por qu.
Hemos visto que todo nmero a puede ser escrito de la forma: a = 4n, a = 4n + 1, a = 4n + 2 o
a = 4n + 3, donde n es un entero.
Deduzca que a2 se escribe solamente de la forma 4p o 4p + 1.
Luego deduzca que una suma a2 + b2 puede ser escrita nicamente de la forma 4k, 4k + 1 o
4k + 2, pero nunca de la forma 4k + 3.
Remark. En este problema, solamente los restos de la divisin por 4 intervienen para concluir que
el resto de a2 + b2 jams es igual a 3.
3 Los

problemas tratados inicialmente se resuelven identificando el mdulo y la clase de ciertos nmeros.

45

La tabla siguiente resume lo esencial:

Restos de b
0
1
2
3

Restos de a
Restos de a2
\
Restos de b2
0
1
0
1

0
1
0
1

1
2
1
2

0
1
0
1

1
2
1
2

Para hacer aparte este trabajo sobre los restos, en lugar de decir, por ejemplo que a = 4n + 3
(lo que muestra que 3 es el resto), escribamos a 3
(md 4), que se lee a es congruente con 3
mdulo 4. As, el proceso anterior se reescribe:

y de la misma forma

a2 0

(md 4) o a2 1

(md 4),

b2 0

(md 4) o b2 1

(md 4)

Se debe demostrar que nunca se obtendr


a2 + b2 3

(md 4)

Cmo obtener este resultado manipulando las congruencias? Admitiremos ac (se probar en
seguida) que las congruencias son compatibles con la suma.
As, se puede sumar y escribir, por ejemplo, que si
a2 0

(md 4) y b2 1

(md 4), entonces a2 + b2 0 + 1 1

(md 4)

Al examinar todos los casos, se obtiene el resultado deseado.


Remark. A nuestro nivel, las congruencias son principalmente una herramienta para simplificar el
lenguaje de la divisibilidad y sus clculos. Estas no permiten probar un resultado que resista a las
tcnicas aritmticas estudiadas anteriormente. Y de igual manera, cualquier demostracin hecha
con congruencias puede ser reescrita en trminos de divisibilidad: en lugar de escribir una igualdad
del tipo ax b
(md m), se escribe ax b es divisible por m.

Actividad 2. Los das de la semana.


Problem. Se numeran los 365 das del ao, asignando 1 al 1 de enero, 2 al 2 de enero, etc. hasta
el 31 de diciembre al cual le corresponde el nmero 365; as, hay una correspondencia entre los das
del ao y los nmeros naturales del 1 al 365. Dado que el 1 de enero de 2010 fue viernes
1.
a) Cul es la forma general de los nmeros que corresponden a los das viernes?
b) Cmo se puede saber si un da cuyo nmero es n es un mircoles? y para reconocer si
es jueves?
46

2. n y p son dos enteros naturales comprendidos entre 1 y 365. Explique por qu las dos proposiciones (P) y (Q) siguientes son equivalentes: (P) Los das de nmeros n y p caen en un
mismo da de la semana. (Q) n y p tienen el mismo residuo en la divisin por 7.4
3. Los das n = 145 y p = 292 caen en un mismo da de la semana?

Actividad 3. Restos de Potencias de un entero en una divisin


A veces es importante analizar los restos de una sucesin de potencias de un nmero con respecto
a un mdulo dado. Por ejemplo
Problem. Los residuos Rn de las potencias de 3 mdulo 7 tienen el siguiente comportamiento:
n
3n
Rn

1
3
3

2
9
2

3
27
6

4
81
4

5
243
5

6
729
1

7
2187
3

8
6561
2

9
19683
6

10
59049
4

11
177147
5

12
531441
1

13
1594323
3

1. Cul el comportamiento que se puede conjeturar de Rn ?


2. Demostrar que el resto de la divisin por 7 de 36 es 1.
3. Deduzca que para todo n: Rn+6 = Rn .
4. Qu propiedad deben cumplir m y p para que Rm = Rp ?
5. Calcule eficientemente R100 , R200 , R300 .

1.

Congruencias mdulo m.

1.1.

Definiciones Equivalentes.

Definition. m es un nmero natural.5 Se dice que dos nmeros a y a0 son congruentes mdulo m
si tienen el mismo residuo en la divisin euclidiana por m, o equivalentemente, si a a0 es divisible
por m.
Demostraremos la equivalencia enunciada en la definicin.
Suponga que a y a0 tienen el mismo residuo en la divisin por m, a = mq + r y a0 = mq 0 + r.
Al restar se obtiene a a0 = m(q q 0 ) con q q 0 Z por lo que a a0 es divisible por m.
Suponga que la diferencia a a0 es divisible por m. Existe un k Z tal que a a0 = km, o
equivalentemente a = a0 + km. Dividimos a0 por m, resulta a0 = mk 0 + r0 con 0 r0 < m.
Entonces a = mk 0 + r0 + km = m(k + k 0 ) + r0 con 0 r0 < m. Esta ltima igualdad prueba
que r0 es tambin el residuo de la divisin de a por m, por lo que tanto a como a0 tienen el
mismo residuo en la divisin por m.
4 Por
5m

tanto, n y p son congruentes mdulo 7.


es llamado el mdulo de la congruencia.

47

...
...
...

Notacin y vocabulario: Siempre que los enteros a y a0 sean congruentes mdulo m, se escribe
a a0
(mod m) y esto se lee: a es congruente a a0 mdulo m.
Example.
(mod m) siempre que r sea el resto de la divisin de a por m.

ar
31 3

(mod 7).

3 2

(mod 5).

9 1

(mod 10).

Remark. Con a0 = 0, se obtiene:


a es divisible por m equivale a a 0

1.2.

(mod m)

Reglas de clculo de las Congruencias.

1. Cualquiera que sea el mdulo, a a.


2. Si a b
(mod m) y si b c
(mod m), entonces a c
(mod m). Esta es la traduccin al lenguaje de congruencias de: En la divisin por m, si a y b tienen el mismo residuo,
y si b y c tienen el mismo residuo, entonces a y c tienen el mismo residuo.
3. Si a b

(mod m) y si a0 b0

a) a + a b + b0

(mod m).

(mod m).

b) a a b b
0

c) aa bb

(mod m), entonces:

(mod m).

Demostracin. Si a b
(mod m) y a0 b0
(mod m), entonces a b = km y a0 b0 =
0
0
0
k m, entonces al sumar se obtiene (a + a ) (b + b ) = (k + k 0 )m con k + k 0 Z, que es divisible
por m.
El mismo tipo de demostracin se aplica para la resta.
Para la multiplicacin, se parte de a = km + b y a0 = k 0 m + b0 , y al hacer la multiplicacin se
obtiene: aa0 = bb0 + m(bk 0 + b0 k + kk 0 m) entonces aa0 bb0 es divisible por m.
Caso particular: Dado que para todo entero x, x x, la aplicacin de esto al resultado anterior
prueba que si a b entonces:
a + x b + x, a x b x, ax bx.
4. Si a b

(mod m), entonces ap bp

(mod m) para todo natural p.

La demostracin es inmediata por recursin sobre el natural p, utilizando la tercera propiedad


del numeral anterior.

48

Cmo utilizar las propiedades de las congruencias?


La idea central de las propiedades de las congruencias es la siguiente: Para sumar, restar, multiplicar, las congruencias obedecen las mismas leyes
que las igualdades.
En particular:
Es posible sumar, restar, multiplicar, miembro a
miembro de congruencias del mismo mdulo.

Example.
3+4 1 (mod 6) implica 4 13
2 (mod 6).
9 3 (mod 6) implica 9x
3x (mod 6) para todo x Z.

x 2 (mod 5) y y 3 (mod 5) implica que x + y 5 (mod 5) y por


tanto x + y 0 (mod 5); tambin,
xy 2 3 1 (mod 5).
10 1 (mod 9) implica 10n
1 (mod 9), para todo n N.

Por qu no es posible simplificar un factor


comn ?
Porque, por ejemplo 2 7 6 (mod 8). Pero, si se pudiera simplificar esta congruencia por 2, se obtendra 7 3 (mod 8), lo
cual es falso dado que 7 3 no es divisible por 8. Sin embargo, en ciertos casos la
simplificacin por un factor comn x puede
ser efectuada: siempre que x y m (el mdulo
de la congruencia) sean primos relativos, entonces la simplificacin por x conduce a una
congruencia vlida.

2.

1. Escriba dos congruencias mdulo m de manera ms simple:


Si x a (mod m) y a0 = a km con k Z, entonces x a0

(mod m).

Demostracin. a0 = a km con k Z implica a a0


resultado.

(mod m), y de all se sigue el

Example. La congruencia x 16

(mod 7).

(mod 7) implica x 2

m
Remark. Todo entero a es congruente mdulo m a un entero x tal que |x| < . En efecto, si
2
r es el resto de la divisin de a por m, entonces:
m
m
, entonces x = r; y si
r < m, entonces x = r m.
2
2

Example. Si a 25 (mod 6), entonces es posible escribir a 1


1 (mod 6) para todo n N.

49

As, se puede enunciar:


Siempre que x y m sean primos relativos, ax
a0 x (mod m) implica a a0 (mod m).
En efecto, ax a0 x (mod m) implica ax a0 x =
(a a0 )x = km con k Z. Como m divide a
(a a0 )x y es primo relativo de x, por el Teorema
de Gauss, m divide a a a0 y por lo tanto a
a0 (mod m).

Aplicacin de las Reglas de Clculo de las Congruencias.

si 0 r <

(mod 6). Adems an

Example. Si a 29

(mod 6), entonces es posible escribir a 1 (mod 6). Adems


(
1 (mod 6)
si n es un nmero par.
n
a
1 (mod 6) si n es un nmero impar.

2. Un caso donde una congruencia implica una igualdad.


Si k k 0

(mod m) con 0 k < m y 0 k 0 < m, entonces k = k 0 .

Demostracin. k k 0 = qm pero |k k 0 | < m por lo que q = 0.

3.

Resolucin de la ecuacin ax b (mod m)

Sean a y m dos nmeros naturales, m > 0. Nos interesa la resolucin en Z de la ecuacin de


incgnita x, ax b (mod m).
Theorem 8. Para que la ecuacin ax b (mod m) admita una solucin, es necesario y suficiente
que el MCD g de a y m divida a b. En particular, la ecuacin tiene solucin si a y m son primos
relativos.
Demostracin. Todas las congruencias siguientes son mdulo m.
1. Si la ecuacin tiene una solucin x0 entonces ax0 b, por lo que ax0 b es divisible por m,
ax0 b = mq con q Z. Dado que g divide a a y m, debe dividir tambin a b.
2. Si g divide a b, entonces b = gq con q Z. Pero g = au + mv, con u, v Z. Entonces
b = auq + mvq y as b auq 0, b auq y uq es una solucin.
Corollary. Si x0 es una solucin de ax b
m
como x0 + k , con k Z.
g

(mod m), todas las soluciones pueden ser escritas

Demostracin. Suponga que ax0 b y ax1 b, se cumple ax1 ax0 0, entonces a(x1 x0 ) = mq
a
m
con q Z. Defina a0 = y m0 = , entonces a0 (x1 x0 ) = m0 q. Pero a0 y m0 son primos entre s,
g
g
por lo que por el Teorema de Gauss, m0 divide a x1 x0 , es decir x1 x0 = m0 k.
m
Recprocamente, es fcil demostrar que si x0 es solucin, todos los nmeros de la forma x0 + k
g
tambin lo son.

50

Nota: Si a < 0,
se
puede
reducir
al
caso positivo
tomando
la
congruencia
equivalente (a)x
b (mod m).
Nota: todas las
congruencias
escritas
son
mdulo m

Para mayor comprensin.


Cmo obtener en la prctica congruencias ms simples?
De acuerdo a las secciones anteriores, dada una congruencia x a (mod m), es posible
disminuir el valor de a quitando a a mltiplos de m.
Example 9. Si x 14023 (mod 7), es posible escribir inmediatamente x 23 (mod 7)
dado que 14000 es visiblemente un mltiplo de 7. Adems es posible pasar de 23 a 2
restando 3 7, as x 2 (mod 7).

Ejercicios resueltos.
Exercise 10. Demostrar que un nmero es divisible por un nmero dado.
Demuestre que 671800 1 es divisible por 6.
Solucin comentada.
800
Se debe demostrar que 671 1 0 (mod 6). Pero: 5 1 (mod 6)
671 = 666 + 5 entonces 671 5 (mod 6), de y: 5800 (1)800 1 (mod 6)
donde: 671800 1 5800 1 (mod 6).
por lo que: 671800 1 1 1
800
El nmero 5
sigue siendo difcil de analizar.
0 (mod 6).
Exercise 11. El objetivo de este ejercicio resuelto es calcular el residuo de la divisin por 7 del
nmero 247349 .
1.
a) Verifique que 247 2 (mod 7).
b) Verifique que 23k 1, 23k+1 2, 23k+2 4
2. Deduzca el resto de la divisin por 7 de 247

349

(mod 7).

Solucin Comentada.
1. a) 247 = 7 35 + 2, por lo que 247 Resulta que 2349 2 (mod 7).
2 (mod 7).
En definitiva:
b) 23 = 8 = 7 + 1, por lo que 23
247349 2 (mod 7)
1 (mod 7). De aqu se deduce que:

k
23k = 23 entonces23k 1k 1 (mod 7).
lo cual implica que el resto de la divisin por
23k+1 = 23k 2 entonces23k+1 1k 2
7 de 247349 es 2.
2 (mod 7).
es
posible
escri23k+2 = 23k 22 entonces23k+2 1k 22 Remark. Cuando
bir
a

1
(mod
m)
se
deduce
que
4 (mod 7).
n
n
a

1
(mod
m)
dado
que
1
=
1.
2. Dado que 247 2 (mod 7), se tiene
que 247349 2349 (mod 7). Pero 349 = Esto permite en la investigacin de restos,
3 116 + 1, entonces 2349 = 23116+1 que reemplazar los nmeros dados por nmeros
mucho ms pequeos.
es de la forma 23k+1 con k = 116.
51

Trabajo Dirigido: Criterios usuales de divisibilidad.


Aplicaremos los conceptos y tcnicas de congruencias para calcular los restos de la divisin para
ciertos nmeros pequeos. Recordemos que todo nmero natural a puede ser escrito:
a = an 10n + an1 10n1 + + a2 102 + a1 10 + a0
donde cada ai representa un nmero natural comprendido entre 0 y 9.6
Example. 3602 = 3 103 + 6 102 + 0 10 + 2.
1. Criterio de Divisibilidad por 9
a) Justifique las afirmaciones siguientes:
10 1 (mod 9) y, para todo natural n, 10n 1 (mod 9).
Si a = an 10n + an1 10n1 + + a2 102 + a1 10 + a0 entonces
a an + an1 + + a2 + a1 + a0

(mod 9)

Deduzca que un nmero natural es divisible por 9 siempre que la suma de sus cifras
es mltiplo de 9, y no es divisible por 9 en caso contrario.
2. Criterio de Divisibilidad por 3.
Haciendo un anlisis similar al anterior, pruebe el criterio siguiente: un nmero natural es
divisible por 3 siempre que la suma de sus cifras es mltiplo de 3, y no es divisible por 3 en
caso contrario.
3. Criterio de Divisibilidad por 5.
Pruebe el criterio siguiente: un nmero natural es divisible por 5 siempre que su ltima cifra
(unidades) es 0 5, y no es divisible por 5 en caso contrario.
4. Criterio de divisibilidad por 4, por 25.
Pruebe los criterios siguientes: Un nmero natural es divisible por 4 (respectivamente por 25)
siempre que el nmero formado por las dos ltimas cifras es divisible por 4 (respectivamente
por 25), y no es divisible por 4 (respectivamente por 25) en caso contrario.
5. Criterio de divisibilidad por 11.
a) Justifique que:
(
n

10

1 (mod 11)
1 (mod 11)

si n es un nmero par.
si n es un nmero impar.

b) Analizando de forma similar a 1. pruebe el siguiente criterio: un nmero natural es


divisible por 11 siempre que la diferencia entre la sumas de cifras pares y cifras impares
es mltiplo de 11, y no es divisible por 11 en caso contrario.
6 Los nmeros {0, 1, 2, 3, 4, 5, 6, 7, 8, 9} son llamados dgitos, haciendo referencia a los 10 dedos de la mano; una
vez que un dgito es colocado en una posicin dentro del nmero (por ejemplo, en las unidades, decenas, centenas,
etc.) recibe el nombre de cifra.

52

c) Aplicacin. El nmero 4792178 es divisible por 11?


6. Un Criterio de divisibilidad por 7.
a) Sea n un nmero natural de cuatro cifras, con la convencin de que la primera cifra de la
izquierda puede ser cero, o las primeras dos, o las primeras tres. As, con esta convencin,
los nmeros 0352, 0061, 0003, son nmeros de cuatro cifras. Se denota por u al nmero
formado por las dos primeras cifras de la izquierda mientras que v es el nmero formado
por las dos cifras de la derecha.
Escribiendo n = 102 u + v, demuestre que n es divisible por 7 siempre que 2u + v es
divisible por 7, y no es divisible en caso contrario.
b) Sin hacer la divisin, diga si los nmeros 343, 8751, 2247, son o no divisibles por 7.
7. Divisibilidad por 37.
Divida 1000 por 37, deduzca un criterio de divisibilidad por 37.

Problemas.
1. Para cada uno de los valores de a, encuentre un entero x tal que x a (mod 7), con 0 x < 7:
a) a = 36.
b) a = 184.
c) a = 3.
d ) a = 7006.
e) a = 4901.
2. Si k 1 (mod 4) con quin es congruente 6k + 1 mdulo 4?
3. Sin hacer uso de calculadora, verifique las siguientes congruencias:
a) 155 35 0 (mod 12).
b) 910 510 0 (mod 7).
4. Resuelva en Z, cada una de las ecuaciones siguientes:
a) x + 3 4 (mod 7).
b) x + 13 3 (mod 7).
5. Resuelva en los enteros, el siguiente sistema de ecuaciones:
(
x 2 (mod 5) , x > 0
x + 2 1 (mod 7) , 100 x < 125
6. Calcule el resto de la divisin de a por b en cada caso:
53

a) a = 8931 y b = 3.
b) a = 57294 y b = 5.
c) a = 3 994 2 4123 y b = 11.
7.
a) Demuestre que 52 1 (mod 13); 54 1 (mod 13).
b) Si k es un natural, demuestre que 54k 1 (mod 13).
8. Encuentre el resto de la divisin por 7 de 3759 82916 y 82916 3759 .
9. Encuentre los naturales x e y menores que 15, tales que x y + 9 (mod 11).
10. Encuentre el resto de la divisin por 7 de 3759 82916 y 82916 3759 .
11. Resolver los sistemas de ecuaciones:
a) x 1 (mod 2) y x 1 (mod 3).
b) x 3 (mod 5) ; x 5 (mod 7) y x 7 (mod 11).
c) 2x 1 (mod 5); 3x 2 (mod 7) y 4x 3 (mod 11).
12. Cules son los ltimos dos dgitos de 713218 ?
13. Cul es el resto de dividir 52010 por 11?
14. Encuentre el resto de la divisin euclidiana por 11 de los nmeros siguientes: 1215 , 107 , 7815 ,
1312 ,(2)19 .
30

15. Encuentre el resto de cada uno de los nmeros 706+1409; 21920 ; (36) ; 835 +640 ; 12517 3012 ,
al dividirlos por 7.
16. a) Cul es el resto de la divisin por 19 del nmero 57383114 ?, b) Cul es el resto de la
divisin por 7 de 912342002 ?
17. Demuestre cada una de las siguientes congruencias:
a) 5n 1 (mod 4).
b) 292n 1 (mod 6).
c) 14n 1 (mod 13).
d ) 12n 1 (mod 13).
18. a) Verificar que 999 es divisible por 27. b) Demostrar que 103n 1 (mod 27). c) Cul es el
resto de la divisin de 10100 + 10010 por 27?
19. Divisin por 111.
a) Demuestre que si se divide un entero n por 111, se obtiene lo mismo que al dividir 1000n
por 111.
54

b) Deduzca que los nmeros 108 + 104 + 1 y 1010 + 108 + 1 son divisibles por 111.
20. Probar que para todo n, 22n 1 (mod 3); 23n 1 (mod 7) ; 24n 1 (mod 15).
21. Encuentre el conjunto de nmeros enteros n tal que 2n + 27 0 (mod 7).
22. Encuentre todos los nmeros n, 1 n 25, tal que n2 + 15n + 122 0 (mod 6).
23. Encontrar todos los n enteros positivos tales que 1848 1914 (mod n).
24. Demuestre que para todo natural n, 32n 2n es divisible por 7.
25. Demuestre que para todo natural n, 5n3 + n es divisible por 6.
26. Demuestre que si n no es mltiplo de 7, entonces n6 1 es mltiplo de 7.

27. Demuestre que para todo natural n, n n2 + 5 es divisible por 6.
28. Demuestre que para todo natural n: n7 + 6n 0 (mod 7).
29. Demostrar que para cualquier natural n, 5n3 + n es divisible por 6.
30. Demostrar que si n es congruente con 4 mdulo 9, entonces n no puede escribirse como suma
de tres cubos.
31. Demostrar que ningn nmero cuadrado perfecto tiene como ltimo dgito 2, 3, 7 u 8.
32. Cul puede ser el ltimo dgito de una cuarta potencia?
33. Demostrar que la diferencia de dos cubos consecutivos nunca es divisible por 3.
34.
a) Sea n un entero. Encuentre todos los restos posibles de la divisin de n2 por 8, luego los
de la divisin de 2n2 por 8.
b) Sean x e y dos naturales, encuentre los restos posibles de la divisin de 2x2 + y 2 por 8.
c) Deduzca de la ecuacin anterior que siendo x e y naturales, la ecuacin 2x2 + y 2 = 5 no
tiene solucin en los enteros.
5

35. Sean a y b dos naturales. Demostrar que (a 2b) a5 (mod b).


36. Sean a y b enteros, tales que a + 5b y 5a b, son ambos divisibles por 2002. Pruebe que a2 + b2
es tambin divisible por 2002.
3

37. Sean a y b dos enteros. calcular (a + b) y deduzca que (a 2b) es congruente con a5 mdulo
3.
3

38. Sean a y b dos enteros. Calcular (a + b) y deducir que (a + b) a3 + b3 (mod 3).

55

39. DIVISIBILIDAD.
Objetivo: Demostrar que para todo natural n, el nmero An = n4n+1 (n + 1) 4n + 1 es
divisible por 9.
Presentamos, tres rutas de solucin diferentes e interesantes:

56

Primera solucin:
1. Para probar que un nmero An es divisible por 9, una primera idea, es probar que
An 0 (mod 9)
es decir, probar que

n4n+1 (n + 1) 4n + 1 0 (mod 9)

Para esto, debemos calcular en primer lugar los restos de cada uno de los trminos dados al dividirlos
por 9; para comenzar, calculamos el resto de 4n (el de 4n+1 se deduce fcilmente). Denotamos por
rn al resto de la divisin de 4n por 9.
a) Calcular r0 , r1 , r2 , r3 , r4 .
b) Dado que r3 1 (mod 9), pruebe que:
4n 1 (mod 9) entonces n = 3p;
4n 4 (mod 9) entonces n = 3p + 1;
4n 7 (mod 9) entonces n = 3p + 2.
c) Pruebe que, dado los tres casos anteriores, An 0 (mod 9).
p
Sugerencia: Utilice el hecho que 43p+k = 43 4k y 43 1 (mod 9).
2. Redacte la solucin.
Segunda solucin.
1. Seguimos la misma idea de probar que An 0 (mod 9). En esta ocasin la factorizacin es muy
til. As en la expresin n4n+1 (n + 1) 4n + 1, pueden factorizarse los primeros dos trminos y
verificar que An 0 (mod 9) es equivalente a
4n (3n 1) 1 (mod 9)
2. Como 3 es un divisor de 9, resulta ventajoso escribir n en la forma 3p , 3p+1, o 3p+2. Pruebe que
para cada uno de los casos anteriores, An 0 (mod 9). Utilice la sugerencia dada anteriormente.
3. Redacte la solucin.
Tercera solucin.
1. Podemos preguntarnos si una recurrencia puede conducirnos a una solucin. La verificacin para
n = 0 no presenta dificultad. Tomamos como hiptesis de recurrencia An 0 (mod 9), es decir,
n4n+1 (n + 1) 4n + 1 0 (mod 9). Debemos entonces demostrar que, bajo esta hiptesis
An+1 = (n + 1)4n+2 (n + 2) 4n+1 + 1 0 (mod 9)
Sin embargo, probar que la ltima expresin se cumple no es fcil, por tal razn es ms conveniente
demostrar que An+1 An 0 (mod 9).
a) Justifique el por qu An+1 An 0 (mod 9), dada la hiptesis de recurrencia, demuestra
que An+1 0 (mod 9).
b) Pruebe que An+1 An = 4n 9q , donde q es un entero.
2. Redacte la solucin.

57

40. Resuelva en Z, cada una de las ecuaciones siguientes:


a) 3x 4 (mod 7).
b) 9x 3 (mod 4).
41. RESOLUCIN DE LA ECUACIN DIOFNTICA ax+by = c donde x e y son dos enteros.
Objetivo: Resolver en Z la ecuacin 3x 5y = 2 utilizando congruencias.
1. Se trata de encontrar el conjunto de soluciones enteras de la ecuacin dada. Supongamos que
existe una solucin (x, y) de la ecuacin. Las congruencias estn relacionadas estrechamente con
los restos, a saber, que a = bm + r implica a b (mod m). La ecuacin 3x 5y = 2 equivale a
3x = 5y + 2 de donde se deduce que 3x 2 (mod 5), es decir, el resto de la divisin de 3x por 5 es
2. Esto nos conduce a estudiar los restos de 3x en la divisin por 5.
a) Cules son los restos posibles de la divisin de x por 5? Deduzca los restos de 3x en la divisin
por 5.
b) En la divisin por 5, para qu restos de x, el resto de 3x es igual a 2?
c) Qu se puede concluir para x? Qu se puede deducir para y?
2. Hemos visto que si las soluciones de x e y existen, toman valores como los encontrados anteriormente. Pero recprocamente, estos valores encontrados de x e y son soluciones de la ecuacin?,
lgicamente, nada nos lo asegura, usted debe verificarlo.
Resuelva el recproco.
3. Redacte una solucin.
42. Encuentre todos los pares de enteros positivos (x, y) tales que 4 (x + 3) = 3y.
43. Encuentre todo los pares de enteros positivos tales que 3 (x + 3) = 5 (y + 3).

44. Sea n un natural y a = n n2 + 5 :
a) Demostrar que a es divisible por 2.
b) Demostrar que a es divisible por 3.
c) Qu puede deducir de los dos literales anteriores?
45. Sea n un natural y a = n (n + 1) (n + 5). Pruebe que a es divisible por 6.
2
46. Sea n un natural y a = 5 n2 + n . Pruebe que a es divisible por 20.
47. Sea n un entero. Pruebe que n (2n + 1) (7n + 1), es divisible por 6.
48. Sea n un entero. Pruebe:
a) n (n + 1) (n + 2) (n + 3) es divisible por 24.
b) n (n + 1) (n + 2) (n + 3) (n + 4) es divisible por 120.
49. El conjunto {Un } est definido para todo entero n por: Un = 5n3 + n. Demostrar que para
todo entero natural n, Un es divisible por 6.
58

50.
a) Demostrar que para todo entero positivo n, 23n 1 es divisible por 7.
b) Deducir que 23n+1 2 es un mltiplo de 7 y que 23n+2 4 es mltiplo de 7.
c) Determine los restos de la divisin por 7 de las potencias de 2.
51. El conjunto{Un } est definido para todo entero n por: Un = 32n 2n . Demostrar que para
todo n, Un es divisible por 7.
52. Sean x e y dos enteros, resuelva la ecuacin 3x + 4y = 1.
53.
a) Demostrar que para todo entero n, n2 es congruente con 0, 1 4, mdulo 8.
2

b) Resuelva en Z la ecuacin (n + 3) 1 0 (mod 8).


54. Demuestre que si a, b y m son divisible por un mismo entero c, entonces
a b (mod m)

b 
m
a

mod
c
c
c

55.
a) Demostrar que 35 1 (mod 11).
b) Deduzca que para todos los enteros positivos k y r: 35k+r 3r (mod 11).
c) n es un entero positivo, encuentre los restos posibles de la divisin de 3n por 11.
d ) Para qu valores del enteros positivo n, 3n + 7 es divisible por 11?
56. Determine los enteros n, tales que 2n 1 es divisible por 9.
57.
a) Demostrar que la suma de tres enteros consecutivos es divisible por 3.
b) Demostrar que la suma de los cubos de tres enteros consecutivos es divisible por 9.
58.
a) Resolver en Z Z la ecuacin: 5p 3q = 1.
b) Deduzca que las soluciones en Z del sistema: x 1 (mod 5) y x 2 (mod 3).
59.
a) Determine el conjunto E1 de enteros x tales que el nmero n = x2 + x 2 es divisible
por 7.
b) Determine el conjunto E2 de enteros x tales que el nmero n = x2 + x 2 es divisible por
3.

59

c) Deduzca el conjunto E de enteros x tales que n es divisible por 42. Cul es el menor
elemento positivo de E.
60. Determine todos los enteros n tales que n2 3n + 6 es divisible por 5.
61. Sea n un natural positivo, tal que n 2.

a) Demuestre que n n4 1 es divisible por 5.
b) Determine los enteros positivos p para los cuales np y np+4 tienen la misma cifra de las
unidades.
62. Nos proponemos encontrar el conjunto de enteros positivos n tales que 43 divide a n2 + n + 41.
a) Pruebe que n2 + n + 41 0 (mod 43) es equivalente a n2 + n 2 0 (mod 43).
b) Factorice n2 + n 2 . Deduzca el conjunto buscado.
63. Estudie segn los valores de n, el resto de la divisin de n7 + 4n + 1 por 8.
64. Demostrar que para todo entero positivo n : 4n + 6n 1 0 (mod 9).
65.
a) Sea a un entero no mltiplo de 3. Demostrar que a3 1 (mod 9) o a3 1 (mod 9).
b) Sean a, b y c son tres enteros; demostrar que si a3 + b3 + c3 es divisible por 9 entonces
al menos uno de ellos es divisible por 3.
66. Resuelva en Z la ecuacin: 3x2 + 4x 0 (mod 21).
67. Encuentre los enteros positivos n tales que 5n + 4n + 2 0 (mod 7).
68. Demostrar que para todo entero positivo n, 32n+1 + 2n+2 0 (mod 7).
69. Demostrar que para todo entero positivo n 2, 55n+2 + 35n1 7 0 (mod 11).
70. Para todo entero n 1 , se define Un = 1 + 3 + 32 + ... + 3n1 :
a) Demostrar que si Un 0 (mod 7) entonces 3n 1 0 (mod 7).
b) Recprocamente, demuestre que si 3n 1 0 (mod 7) entonces Un 0 (mod 7).
c) Deduzca los valores de n para los cuales Un es divisible por 7.
71.
a) Estudie segn los valores de n, el resto de la divisin de 7n por 10.
b) Haciendo A = 1 + 7 + ... + 7n , para todo natural n. Encuentre la cifra de las unidades
de A.
72. Haciendo A = 109n + 27 106n + 2 103n + 1
a) Estudie segn valores de n, el resto de la divisin de 10n por 111.
60

b) Deduzca el resto de la divisin de A por 111.


73. Para A = 109n + 27 106n + 2 103n + 1, supongamos:
a) n impar, demuestre que A es divisible por 7, por 11 y por 13.
b) n par, demuestre que A tiene el mismo resto en la divisin por 7, por 11 y por 13.
c) Utilizando los dos literales anteriores, encuentre, segn los valores de n, el resto de la
divisin de A por 1001.
74. Haga lo que se indica:
a) Verificar que el nmero 7 divide a los nmeros: 26 1; 36 1; 46 1; 56 1.
b) Sea n un entero positivo y Sn = 2n + 3n + 4n + 5n . Demostrar que Sn+6 Sn es divisible
por 7.
c) Sean n un entero positivo y r uno de los restos de la divisin por 6. Demostrar que
Sn Sr (mod 7).
d ) Encuentre los valores de n para los cuales Sn es divisible por 7.
e) Suponga que Tn = 100n + 101n + 102n + 103n . Demostrar que Sn Tn (mod 7) y
deduzca los valores de n para los cuales Tn es divisible por 7.
75. Ana dice que 27, 182, 818, 284, 590, 452 es divisible por 11. Beto dice que no. Quien tiene la
razn?
76. Cules dgitos pueden ser colocados en lugar de x e y en el nmero 30x0y03 en ese orden,
para que el nmero sea divisible por 12?
77. Un palndromo es un nmero que se lee igual hacia delante y hacia detrs. Por ejemplo: 22,
1331 y 935686539 son palndromos.
a) Demostrar que todo palndromo de 4 dgitos es divisible por 11.
b) Qu ocurre con los palndromos de seis dgitos?
78. Sean x, y e z, enteros:
a) Cules son los restos posibles de la divisin de x2 3y 2 por 4.
b) Existen los enteros x, y e z tales que x2 3y 2 4z 2 = 3?
79. Sean x, y, z, enteros positivos tales que 7 divide a x3 + y 3 z 3 , pruebe que alguno de ellos es
un mltiplo de 7.
80. Para x e y enteros. Resuelva el sistema: 3x + y 1 (mod 6) y x y 3 (mod 6).
81. Resuelva el sistema: 3x 1 (mod 5) y 5x 2 (mod 6).
82. Sean x, y e z, enteros tales que: x2 + y 2 = z 2 . Demostrar las siguientes proposiciones:
a) Uno de los nmeros x o y es mltiplo de 3.
b) Uno de los nmeros x o y es mltiplo de 4.
c) Uno de los nmeros x , y o z es mltiplo de 5.

61

Parte IV

Los nmeros primos


1.

Definicin. Los nmeros primos

En todo este apartado cuando nos refiramos a nmeros enteros se entender que son positivos;
cuando nos refiramos a divisores o mltiplos de un nmero lo entenderemos como mltiplos o
divisores positivos.

1.1.

Definicin y existencia

Definition. Un nmero primo es un entero estrictamente superior a 1 que no admite ms divisores


que el nmero mismo y la unidad.
Definition. Cuando un nmero p es primo el conjunto de divisores se reduce al conjunto D =
{1, p}. Un nmero no primo n se dice que es un nmero compuesto; en tal caso el nmero n admite
otros divisores distintos a los del conjunto D = {1, p}; tales divisores se denominan divisores
estrictos del nmero n; si d un divisor estricto se tiene la desigualdad 1 < d < n.
Definition. Dos nmeros enteros se dir que son primos relativos o coprimos si no tienen ms
divisores comunes que la unidad. Por ejemplo son primos relativos los nmeros 15 y 16 pues el
nico divisor comn a ambos es 1. Cuando p es un nmero primo y n es un entero que no es
divisible por p, los nmeros n y p son primos relativos.
Theorem. Todo entero n > 2 admite un nmero primo como divisor.
Demostracin. Slo hay dos posibles casos: o el entero n es primo o bien es un nmero compuesto.
Si n es primo, el divisor primo es el mismo n. Si es compuesto n posee cuando menos un divisor
estricto; ordenemos en orden creciente los divisores de n, 1 < a1 < a2 < < as < n; entonces
el divisor estricto a1 , el menor de todos los divisores estrictos de n, es primo. En efecto si as no
fuera, a1 como nmero compuesto tendra un divisor estricto d, que a su vez sera divisor de n, con
d < a1 , lo que sera contrario al supuesto de que a1 es el menor divisor estricto de n.
En la bsqueda de los divisores estrictos de un nmero n es importante observar que si d es
uno de sus divisores estrictos, entonces n se escribe en la forma n = dd1 , de donde resulta que d1
es tambin un divisor estricto de n; obviamente no es posible que ambos divisores estrictos sean
mayores que n, pues de lo contrario el producto dd1 = n sera mayor que n, lo que es absurdo.
Como consecuencia de esta observacin, para
saber si un nmero es primo basta con reducir la
bsqueda de sus divisores estrictos d, con d 6 n. Por ejemplo,para determinar si 1039 es primo
es suficiente revisar si posee o no divisores menores o iguales a 1039.

62

1.2.

Todos los nmeros primos

Theorem. El conjunto de nmeros primos es conjunto infinito.


Demostracin. Demostraremos que para cualquier nmero primo p, es posible encontrar otro nmero primo p1 que es estrictamente mayor que p. Sea entonces p un nmero primo y construyamos
el entero n = 2 3 5 7 p, producto de todos los primos menores o iguales a p. S ea
ahora p1 = n + 1; por el teorema1.1 el entero p1 posee un divisor primo q. Este divisor primo q no
puede ser ninguno de los primos menores o iguales a p, ya que la divisin de p1 por cualesquiera
de ellos tiene residuo igual a 1; en consecuencia q es un primo diferente a todos los anteriores y es
estrictamente superior a p.

2.
2.1.

Descomposicin en productos de los nmeros primos.


Teorema Fundamental

Theorem. Todo entero n superior o igual a 2, es producto de nmeros primos.


Demostracin. Sea n un entero mayor que 2. Si n es primo n, entonces se expresa como el producto
de un slo trmino. Si n no es primo, segn el teorema 1.1, n admite un divisor primo p1 y por lo
tanto n puede ser escrito en la forma n = p1 q1 . Si q1 es primo el teorema queda demostrado; si q1 es
compuesto entonces q1 puede ser expresado en la forma q1 = p2 q2 , y en consecuencia n = p1 p2 q2 ; de
nuevo si q2 es primo el teorema queda demostrado, de lo contrario puede ser expresado en la forma
q2 = p3 q3 y n = p1 p2 p3 q3 ; dado que la sucesin {qi } es estrictamente decreciente, este proceso debe
terminar en un nmero finito de pasos. As, n se expresa en la forma n = p1 p2 p3 pl .
En la demostracin anterior los nmeros primos que intervienen en la descomposicin pueden
ser iguales, en este caso agrupando estos factores idnticos tendremos la forma ms general de la
descomposicin del nmero natural:
k
1 2
n = p
1 p2 pk

en donde j representan el nmero de veces que aparece como factor el primo pj .


Definition. Descomponer un nmero natural n en producto de nmeros primos p1 , p2 , p3 , . . . , pk ,
k
1 2 3
es expresarlo en la forma denominada cannica: n = p
1 p2 p3 ....pk , con p1 < p2 < p3 < ... < pk
y 1 , 2 , 3 , ..., k enteros positivos no nulos.
Por ejemplo la descomposicin cannica del nmero 28 es: 28 = 22 71 .
Expresar un nmero en su forma cannica permite determinar el nmero de divisores en forma
relativamente sencilla. Para el caso del ejemplo anterior, los divisores del nmero 28 son 20 70 , 21 70 ,
22 70 , 20 71 , 21 71 , 22 71 , es decir 1, 2, 4, 7, 14, 28, que son todos los enteros que pueden ser expresados
en la forma 21 72 , con, 0 1 2 y 0 2 1. As, cada divisor est asociado a un par de
enteros (1 , 2 ) y dado que para 1 hay 3 posibilidades y para 2 hay 2 posibilidades, el nmero de
divisores debe ser 6.

63

2.2.

Investigando los divisores de un nmero natural no primo

En general, la descomposicin de un nmero n en sus factores primos permite determinar el nk


1 2 3
mero de divisores de n. En efecto, si se ha logrado descomponer n en la forma n = p
1 p2 p3 pk ,
los divisores de n son todos los enteros positivos de la forma d = p1 1 p2 2 p3 3 pkk con las condiciones 0 j j para todo j, 1 j k , por lo que a cada divisor est asociada una cadena
(1 , 2 , ...., k ) de enteros. Dado que para cada exponente j del nmero primo pj tenemos j + 1
posibles valores (los valores desde 0 , hasta el valor j ), el total de posibles divisores d de n es
(1 + 1)(2 + 1)(3 + 3) (k + 1)

2.3.

El Mximo Comn Divisor

Otro importante uso de la descomposicin de un nmero en su forma cannica, es la determinacin del Mximo Comn Divisor y el Mnimo Comn Mltiplo de un conjunto de nmeros
dados.
Consideremos por ejemplo los nmeros n1 = 23 32 51 111 , n2 = 24 32 52 72 y n3 = 22 33 52 7. Es claro
que de las potencias de 2, la mxima que es divisor comn a los tres nmeros es 22 , de las potencias
de 3, la que divide a los tres nmeros es 32 , de las potencias de 5, la comn a los tres nmeros
es 51 ; observe adems que las potencias de 7 y las de 11 no son comunes a los tres nmeros en
consecuencia no pueden participar en la composicin del MCD de los tres nmeros. Se deduce de
lo anterior que M CD(n1 , n2 , n3 ) = 22 32 51 . En cuanto a los mltiplos comunes a los tres nmeros
se observa que en ellos la mnima potencia de 2 que debe aparecer es 24 , la mnima potencia de
3 que puede aparecer en los mltiplos de los tres nmeros es 33 , la potencia mnima de 5 que
puede aparecer en los mltiplos comunes es 52 , la potencia de 7 mnima es 72 y todo mltiplo
comn debe contener la potencia mnima posible de 11, 111 . As el MCM de los tres nmeros es
M CM (n1 , n2 , n3 ) = 24 33 52 72 111 .
En general, cuando se dispone de la descomposicin cannica de un conjunto de nmeros
n1 , n2 , . . . , nk el MCD se logra seleccionando, como exponente de cada uno de los factores primos comunes que intervienen en la descomposicin, el mnimo exponente con el que aparece dicho
factor. De manera similar para la determinacin del MCM se escoge para todos los factores primos
que intervienen en las descomposiciones, la mxima potencia posible.

2.4.

LAS POTENCIAS DE UN NMERO Y LA DESCOMPOSICIN


CANNICA.

Un resultado importante, que se deduce de la descomposicin cannica de un nmero, es el


siguiente:
k
1 2
Si en la descomposicin de un nmero n = p
1 p2 ....pk todos los exponentes son pares, entonces
el nmero n es un cuadrado perfecto.
En efecto, si todos los exponentes j son pares, stos pueden ser expresados en la forma j = 2j
2

y en tal caso n = p1 1 p1 2 p1 3 pkk y en consecuencia n es un cuadrado.
El resultado anterior por supuesto puede ser extendido a cualquier potencia; por ejemplo si en
la descomposicin cannica de n, todos los exponentes son mltiplos de 3, el nmero n es un cubo;
si todos los exponentes son mltiplos de 4, entonces n es una potencia cuarta y as sucesivamente.

64

Theorem. Si u y v son dos nmeros primos relativos y su producto uv es un cuadrado, entonces


u y v deben ser cuadrados.
1 2
l
k
1 2
Demostracin. Para demostrarlo supongamos que u = p
1 p2 pk y v = q1 q2 ql y que
uv es un cuadrado. Como u y v son primos relativos no tienen ms factores comunes que 1 y en
n
m
consecuencia es imposible una igualdad del tipo p
m = qn , de donde se deduce que la descompol
1 2
k 1 2
sicin cannica de uv es simplemente el producto p1 p2 p
k q1 q2 ql y dado que uv es un
cuadrado todos los exponentes en tal producto deben ser pares, de donde se obtiene que tanto u
como v deben ser cuadrados. Nuevamente este resultado puede ser extendido a otras potencias, por
ejemplo si uv es un cubo y u y v son primos relativos, entonces tanto u como v deben ser cubos y
as sucesivamente.

3.
3.1.

Nmeros primos y divisibilidad en N


Divisibilidad por un nmero primo

Theorem. Si p es un nmero primo que divide al producto ab de dos nmeros naturales a y b,


entonces p divide al natural a o divide al natural b. Adems si a y b son nmeros primos, entonces
a = p o b = p.
Demostracin. Si p divide al natural a, el teorema se cumple, si p no divide a a entonces p y a son
primos relativos y en consecuencia, de acuerdo al teorema de Gauss, p divide a b. Si adems a y b
son primos, siendo que p debe dividir al nmero a o al nmero b, entonces p = a o p = b.

3.2.

El Pequeo Teorema de Fermat.

Theorem 12. El pequeo teorema de Fermat. Si p es un nmero primo y a es un entero no divisible


por p, entonces ap1 1 es divisible por p.
Demostracin. Consideremos los mltiplos de a: a, 2a, 3a, . . . , (p 1)a y los restos que se obtienen
al dividirlos por p. Sea rk el resto de dividir ka, por p , para k = 1, 2, 3, , (p 1). Los rk son
todos distintos de cero: en efecto, si algn residuo rk fuese cero, ka sera divisible por p; puesto que
p es primo, significara que p divide a a o que p divide a k. Ahora bien, p no divide a a por las
condicin del teorema y tampoco divide a k, puesto que k < p, por lo que los residuos deben ser
todos distintos de cero. Adems, los residuos son todos diferentes: si hubiese dos residuos iguales
rk1 = rk2 , entonces se tendra ak1 ak2 mdulo p, pero siendo que p y a son primos relativos,
entonces necesariamente sera k1 = k2 .
Si escribimos ahora las congruencias en mdulo p : a r1 , 2a r2 , 3a r3 , . . . , (p 1)a rp1
y las multiplicamos miembro a miembro tendremos:
a 2a 3a (p 1)a r1 r2 r3 rp1 , es decir ap1 (p 1)! r1 r2 r3 rp1 .
Puesto que todos los residuos son diferentes y dado que los residuos posibles son 1, 2, 3, . . . , (p1),
sin importar el orden de aparicin de tales residuos, el producto en el miembro derecho de la
ltima igualdad debe ser (p 1)!, de donde se obtiene la relacin: ap1 (p 1)! r1 r2 r3 rp1
(p 1)!. Siendo que p es primo, es primo relativo con cada uno de los nmeros 1, 2, 3, . . . , (p 1); en
consecuencia es vlida la simplificacin, es decir de la relacin ap1 (p 1)! (p 1)!, se obtiene que
ap1 1 (mod p), que es equivalente a la afirmacin (ap1 1) es divisible por p, lo que demuestra
el teorema.
65

Haremos una segunda demostracin del Pequeo Teorema de Fermat haciendo uso de una propiedad del nmero combinatorio, para ello demostraremos dos lemas.
Lemma. Sea p un nmero primo y k un entero con 1 k (p 1), entonces el nmero combinatorio Cpk es divisible por p.
p!
, es decir tenemos Cpk k!(p k)! = p!. Como p es
k!(p k)!
primo y obviamente divide a p!, p divide en consecuencia al miembro izquierdo y por lo tanto debe
dividir a alguno de los factores que intervienen en l. Ahora bien, p no divide a k! puesto que todos
los factores en k! son estrictamente menores que p, por la misma razn p no divide a (p k)! y por
lo tanto es forzoso que p divida al nmero combinatorio Cpk , lo que demuestra el lema.
Demostracin. Observemos que Cpk =

Lemma. Si p es primo entonces np n (mod p).


Demostracin. En el desarrollo del binomio (a+b)p , todos los trminos excepto ap y bp , son divisibles
por p de acuerdo al lema anterior, en consecuencia se tiene (a + b)p ap + bp (mod p). Razonando
por recurrencia se deduce que (a1 + a2 + a3 + + an )p ap1 + ap2 + ap3 + + apn y reemplazando
los ai por 1 se obtiene np n (mod p), lo que demuestra el lema.
Theorem. Segunda demostracin del Pequeo Teorema de Fermat.
Demostracin. De acuerdo al lema anterior ap a (mod p), y tomando en cuenta que p y a son
primos relativos podemos simplificar para concluir que ap1 1 (mod p).

3.3.

Ejercicios de Nmeros Primos.

1. Determinar si los nmeros 24317, 55319, 11303 y 55331 son primos.


2. Es 852 4 el producto de dos nmeros primos?
3. a) n es un nmero natural. Puede ser primo el nmero 5n2 + 7n?
b) Por qu el nmero n2 5n + 6 no es primo, excepto cuando el natural n sea igual a 1 4?
Nota: Las dos preguntas son independientes.
4. Tres tarjetas tienen respectivamente las cifras 1, 1, 3. Se colocan al azar sobre un soporte
formando as un nmero de tres cifras. Cuntos nmeros diferentes se pueden formar? Prueba
que todos son primos.
5. Demuestre que los nmeros 20! + 2; 20! + 3; 20! + 4 y 20! + 5 no son primos.
6. p es un nmero primo, p > 3. Demuestre que p 1 o p 5 es divisible por 6.
Sugerencia: Se pueden utilizar las congruencias.
7. Demuestre que todo nmero primo p, p > 2, verifica una u otra de las congruencias p
1 (mod 4) o p 3 (mod 4).

66

8. p es un nmero primo, mayor o igual que 5.


a) Demuestre que p2 1 es divisible por 3.
b) Demuestre que p2 1 es divisible por 8.
c) Deduzca que p2 1 es divisible por 24.
Sugerencia: Se podrn utilizar las congruencias.
9. p es un nmero primo.
a) Cules son los posibles restos en la divisin de p por 12?
b) Pruebe que p2 + 11 es divisible por 12.
10. p es un nmero primo. p > 4. Se denota a un nmero natural tal que 2 a < p1. Demuestre
que p no divide a a2 1.
11. Descomponer los nmeros en producto de nmeros primos.:
a) 13552.
b) 11737.
12. En cada uno de los casos, descomponer el nmero dado en producto de nmeros primos y
encontrar todos sus divisores:
a) 450.
b) 1352.
13. En cada uno de los casos, explique por qu el nmero dado es un cuadrado perfecto y encuentre
su raz cuadrada.
a) 28 52 132 b) 512 1114 1336 c) 210 34 292
14. Encontrar el mximo comn divisor y el mnimo comn mltiplo de a y b:
a) a = 22 52 13 y b = 23 32 52
b) a = 32 52 11 y b = 23 3 5
15. Encontrar el mximo comn divisor y el mnimo comn mltiplo de a y b:
a) a = 810 y b = 1764.
b) a = 12375 y b = 2970.
16. Cules son los nmeros naturales que nicamente admiten tres divisores?
17. m es un nmero que se escribe 9 10n . Encuentre n de tal manera que m posea 27 divisores.
18. Encontrar un nmero de tres cifras que sea un cuadrado perfecto divisible por 56.
19. Cul es el cuadrado perfecto ms pequeo divisible por 616?

67

20. a) Descomponer 504 en producto de nmeros primos.


b) a y b son dos enteros naturales primos entre s tales que ab = 504. Encontrar a y b.
21. p es un nmero primo y n es un entero natural. Cuntos divisores positivos tiene el nmero
pn ? Cul es la suma de esos divisores?
22. a) Encontrar el conjunto de divisores de 196.
b) Resolver en los naturales la ecuacin x2 y 2 = 196.
c) Repetir a) y b) con el nmero 225.
23. Un entero natural n se dice que es perfecto cuando la suma de sus divisores es igual a 2n.
Son perfectos los nmeros 496 y 8128?
24. Es 401 un nmero primo? Resolver en el conjunto de los naturales la ecuacin x2 y 2 = 401.
25. Dos enteros a y b son tales que a + b = 486 y 100 < a < b. Denotemos por d su MCD.
1) a) Porqu d divide al nmero 486?
b) Porqu d = 2 3 con 0 1; 0 5?
2) Sabemos adems que a y b tienen exactamente 6 divisores comunes, que ab es mltiplo de
10 y que 5 no divide a b.
a) Demostrar que d = 18.
b) Deducir los valores de a y b.
26. El producto de dos enteros naturales a y b (a < b) es 11340. Denotemos por d su MCD.
1) a) Por qu d2 divide a 11340?
b) Por qu d = 2 3 con 0 1 y 0 2?
2) Sabemos adems que a y b tienen 6 divisores comunes y a es mltiplo de 5.
a) Demuestre que d = 18.
b) Deduzca a y b.
27. n es un entero natural no nulo. Denotemos a = n5 n.
a) Por qu a es divisible por 5?


b) a = n n2 1 n2 + 1 ; Por qu a es divisible por 3?
c) Por qu a es divisible por 30?
28. n es un entero, n 1; a = n13 n
a) Probar que a es par.
b) Probar que a es divisible por 13 y por 7.
c) Deducir que a es divisible por 182.
29. a) Demostrar que 2003 es primo.
b) Deducir que para todo entero natural n que no es mltiplo de 2003, el residuo de la divisin
de n2002 por 2003 es igual a 1.
68

30. Demostrar que aparte de 3, 5 y 7, no existen tres nmeros impares consecutivos, tales que los
tres sean primos.
31. a y b son dos naturales y p un nmero primo. Demostrar que si a2 b2 = p, entonces a y b
son consecutivos (es decir, a = b + 1).
32. Encontrar todos los enteros x e y tales que x2 + 6x = y 2 + 47.
33. n es un entero, n 2. Demuestre las proposiciones siguientes:


a) n2 n2 1 n2 4 es divisible por 360.


b) n2 n4 1 n4 16 es divisible por 600.
34. a, a + b, a + 2b son tres nmeros primos dados, a 5.
a) Demostrar que b es un nmero par.
b) i) Demostrar que b es divisible por 3 ii) Demostrar que b es divisible por 6.
35. Demostrar que si la suma de dos nmeros naturales es un nmero primo, entonces los dos
nmeros son primos entre s.
36. Demostrar que si dos nmeros primos, superiores a 3, tienen una diferencia igual a 8, su
semisuma no es un nmero primo.
37. a) Descomponer en R el polinomio x4 + 4 en un producto de dos polinomios de segundo grado.
b) n es un natural Existe algn nmero primo de la forma n2 + 4?
c) Dados dos nmeros naturales n y n0 tales que n n0 = 2. Demostrar que los nmeros
e = n4 + 4 y e0 = (n0 )4 + 4 no son primos entre s.
38. p es un nmero natural; p > 1. La suma de p impares consecutivos puede ser un nmero
primo?
39. a) Descomponer 600 en su producto de factores primos.
b) Encontrar el menor entero natural k tal que 600k es un cuadrado perfecto.
40. n es un entero natural no nulo.
a) Cules son los divisores primos de 10n ? Cmo podemos reconocer a partir de su descomposicin en factores primos que un nmero es una potencia de 10?
b) Para cada uno de los nmeros m siguientes, indicar justificando su respuesta, si existe un
natural k tal que mk es una potencia de 10. Si k existe, encontrar su valor ms pequeo.
i) m = 40 ii) m = 375 iii) m = 160 iv) m = 12800
41. p es un nmero primo, p > 2. Determinar en funcin de p los naturales a y b tales que
a2 b2 = p2 .
42. k es un natural no nulo. Denotemos a = 9k, b = 8k + 6.
Demostrar que para todo k, a y b no pueden admitir ms que dos divisores primos distintos.
Precisar esos divisores.
69

43. p es un nmero primo, p 5.


a) Demostrar que p se escribe como 6k 1 6k + 1 con k entero, k > 0.
b) Deducir que p2 1 es divisible por 24.
44. Encontrar todos los pares (x, y) de nmeros naturales tales que (x 3)(y + 7) = 13(y 2).

70

45. La ecuacin de Pitgoras: X 2 + Y 2 = Z 2 :


Se trata de determinar los enteros naturales x, y, z tales se cumpla la ecuacin x2 + y 2 = z 2 . Geomtricamente es equivalente a determinar los tringulos rectngulos cuyos lados tienen longitudes
enteras. Es clsico el tringulo rectngulo de catetos 3 y 4 y de hipotenusa 5.
La ecuacin de Pitgoras tiene infinidad de soluciones enteras.
El siguiente resultado muestra que la ecuacin pitagrica posee una infinidad de soluciones.
a) Demuestre que para cualesquiera nmeros reales o complejos u, v se cumple que la terna
(x, y, z) con x = 2uv , y = u2 v 2 y z = u2 + v 2 satisface la ecuacin x2 + y 2 = z 2 . Este
resultado permite asegurar que si se eligen nmeros naturales u y v con u > v > 0, la terna
(x, y, z) es una terna pitagrica.
b) Verifique adems que si u y v se escogen de diferente paridad, entonces tambin x y y son de
diferente paridad.
c) Asuma que u y v son primos relativos. Demuestre que x,y son tambin primos relativos, lo
mismo que las parejas (x, z) y (y, z).
d ) Verifique que si (x, y, z) es una terna pitagrica y d es cualquier nmero natural, entonces la
terna (dx, dy, dz) es tambin una terna pitagrica.
e) Demuestre que cuando u y v no son primos relativos, entonces x, y , z tienen un divisor comn
d.
Los siguientes pasos son para verificar que todas las ternas pitagricas se logran en la forma anteriormente descrita.
a) Suponga que x es par, y impar y z impar y adems que x y y son primos entre s. Pruebe que
x y z son primos entre s lo mismo que y y z.



z+y
zy z+y
zy
2
y que los nmeros
y
son
b) Se escribe x = 2q. Verifique que q =
2
2
2
2
naturales estrictamente positivos.
c) Pruebe que

zy z+y
y
son primos relativos.
2
2

d ) Deduzca de lo anterior que existen enteros u y v tales que 0 < u < v con
v2 .

zy
z+y
= u2 y
=
2
2

e) Deduzca que x = 2uv; y = v 2 u2 y z = u2 + v 2 .


f ) Se supone que la terna (x, y, z) es una terna pitagrica y que x y y no son primos relativos.
Sea g el MCD de x y y. Pruebe que g divide a z, y que g divide a z utilizando para ello las
descomposiciones cannicas de z, g, g y z.
g) Se escribe x = x1 g, y = y1 g y z = z1 g. Verifique que la terna (x1 , y1 , z1 ) es pitagrica.
h) Supngase que x1 y y1 son ambos nmeros impares. Deduzca que z12 es de la forma 4m + 2
con m entero. Pruebe que un cuadrado no puede ser de esta forma, y que en consecuencia no
pueden ser x1 y y1 ambos nmeros impares. Este resultado justifica el por qu del supuesto
71
en el numeral 1.

46. Los Nmeros Perfectos.


Definition. Se llama nmero perfecto todo entero natural n cuya suma de todos sus divisores
positivos es igual a 2n.
Example. El 6 y el 28 son nmeros perfectos, dado que los divisores de 6 son 1, 2, 3 y 6, y su
suma es 12; y lo mismo sucede con 28, la suma de sus divisores 1, 2, 4, 7, 14 y 28, es 56.
Theorem. Teorema de Euclides: Si 2n+1 1 es primo, entonces el nmero a = 2n (2n+1 1) es
un nmero perfecto.
Demostracin. Los siguientes pasos permiten demostrarlo:
a) Descomponga a en sus factores primos.
b) Deduzca la lista de todos los divisores de a.
c) Demuestre que la suma de todos los divisores es igual a 2a.
47. El Teorema de Wilson.
Theorem. Teorema de Wilson: Para todo nmero primo p, (p 1)! + 1 es un mltiplo de p.
Los siguientes pasos sirven para lograr la demostracin:
a) Verifique el teorema para p = 2 y p = 3.
b) Cuando p > 3 se escribe la sucesin de nmeros 2, 3, 4, . . . , (p 3), (p 2). Sea a un nmero
de esta sucesin. Verifique que el residuo de la divisin de a por p es igual a a.
c) El residuo de ap2 al dividirlo por p es un nmero de la sucesin, es decir, que ap2 r(mod p)
con r = 2, 3, . . . , (p 2). Sugerencia: Razone por reduccin al absurdo y trate de lograr una
contradiccin utilizando el Pequeo Teorema de Fermat.
Remark. Este resultado es equivalente a afirmar que r 6= 0, r 6= 1, r 6= p 1.
d ) Pruebe que a todo nmero a de la sucesin se le puede asociar un nmero a0 tambin de la
sucesin (distinto de a) de forma tal que aa0 1 (mod p).
e) Pruebe que si a 6= b entonces a0 6= b0 . Otra vez razone por reduccin al absurdo.
p3
congruencias mdulo p, ai a0i 1 (mod p) tales que los
2
nmeros ai y a0i que aparecen son exactamente los nmeros 2, 3, . . . , (p 2).

f ) Deduzca que se pueden escribir

g) Finalmente, multiplicando miembro a miembro todas las congruencias anteriores, deduzca que
(p 2)! 1 (mod p) y enseguida multiplicando por p 1 se obtiene (p 1)! p 1 (mod p),
o equivalentemente (p 1)! 1 (mod p), lo que demuestra el teorema.

72

También podría gustarte